Clat Mock 1-1
Clat Mock 1-1
Clat Mock 1-1
Explanation: Marilyn Monroe was born on 1 June, 1926 in Los Angeles, California, U.S.A and died
unexpectedly due to a Drug Overdose on 4 th August, 1962, at an extremely young age of just 36. She was
the top grossing American actress, Singer and Model in the Hollywood in 1950s and is still remembered
for her comic ‘blonde bombshell’ characters.
She is also widely recognized for her controversial yet famous quotes, particularly on women
empowerment. One of her famous Quote was “Give a girl the right pair of shoes and she’ll conquer the
world”.
42. The International Space Station (ISS), which is currently the world’s only space station with a Human
crew, has how many astronaut on board when fully crewed?
A. Six
B. Seven
C. Three
D. Four
Explanation: The International Space Station located at the distance of 400 km above the Earth’s surface
is a multinational collaboration of Europe, The United States, Russia, Japan and Canada, and it has six
astraunauts on board when Fully Crewed. It is the largest human made-body in the lower earth Orbit
and is often seen with Naked eyes too.
It was launched on 20 November, 1998 and has a massive mass of 360-tonne.
43. The RBI committee appointed in December 2018 to review Economic Capital Framework (ECF) was
headed by which of the former RBI Governor:
Explanation: The Reserve Bank of India (RBI) committee led by former RBI governor Bimal Jalan was
constituted in December 2018 to review Economic Capital framework (ECF) for the Central bank after
the Union Finance Ministry under Arun Jaitely advised RBI to transfer surplus funds to Central
Government.
The panel was supposed to submit its report in April, 2019 but it was delayed due to lack of Consensus
among its members.
44. Which Country’s Supreme court on 14 th June, 2019 officially made Homophobia and transphobia
crimes similar to Racism?
A. Netherlands
B. Sudan
C. Canada
D. Brazil
Explanation: Brazil’s Federal Supreme Court’s 11 judge bench ruled with a majority of 8:3 in favor of
LGBT community and declared Homophobia and Transphobia crimes similar to Racism, until the
country’s legislature passes a separate legislation dealing with LGBT discrimination. Racism was
criminalized in the country in 1989 with imprisonment extending up to a term of 5 years and now
‘Transphobia’ and ‘homophobia’ will also be punishable under the law penalizing racism.
[References:
Homophobia: dislike or hatred feelings for homosexual people (LGBT, i.e. Lesbian, Gay, Bisexual and
Transgender).
45. Sudan which is one of Africa’s largest nation, recently witnessed an international crisis after ending 3
decades old authoritative rule of which President by a military coup led by Sudanese Defence Minister?
Explanation: Omar al-bashir served as the president of Sudan from 30 th June, 1989 to 11th April, 2019.
He was recently overthrown on 11th April, 2019 by a Military-coup led by Sudanese Defence Minister,
Awad Mohamed Ahmed Ibn Auf. Bashir acted as a dictator of Sudan for 30 years, since 1989 when he
came to power by overthrowing the Democratic Government led by Prime Minister, ‘Sadiq-al Mahdi’ by
a Military Coup. He is accused of War Crimes and Crimes against Humanity by the International Court of
Justice in connection to the Conflict in Sudan’s Darfur Province, wherein he is alleged to direct a
campaign of Mass killing, harassment and Rape of Civilians in Darfur.
46. The ‘Namami Gange Programme’ which was approved as a ‘flagship programme’ by the Government
of India in 2014 for reducing pollution and for Conservation and rejuvenation of national river Ganga has
been allocated a budget outlay of ______________ Rupees.
A. 10,000 Crore
B. 15,000 Crore
C. 25,000 Crore
D. 20,000 Crore
Explanation: While presenting the budget in the parliament on 10 th July, 2014, the Union Finance
Minister, Arun Jaitley announced an integrated Ganga development project called ‘Namami Gange’. The
Project had been allocated with a budget outlay of 20,000 Crore rupees spread over the next five years.
As a part of the Project, around 40 industries have been closed down that were situated near the banks
of the River Ganga and were polluting it. Eight of the Indian States have been covered under this Project.
47. Who is appointed as Minister of Agriculture and Farmers welfare in the 2019 newly-elected
government of India under the leadership of Prime Minister Narendra Modi?
A. Rajnath Singh
B. Ram Vilas Paswan
C. Ravi Shankar Prasad
D. Narendra Singh Tomar
Explanation: Narendra Singh Tomar has been appointed as Minister of Agriculture and Farmers welfare.
In addition he will also hold the charge of Ministry of Rural development and Ministry of Panchayati Raj.
He is elected to the 17th Lok Sabha from the constituency of Morena. He was also elected as the member
of Lok Sabha form 2014 to 2019 from the constituency of Gwalior and served as a Minister of Mines and
Minister of Parliamentary affairs in Government of India from 2014-2019.
Option (A), i.e. Rajnath Singh is appointed as Defence Minister, in Government of India.
Option (B), i.e. Ram Vilas Paswan is appointed as Cabinet Minister of Consumer Affairs, Food and Public
Distribution.
Option (C), i.e. Ravi Shankar Prasad is appointed as Union Minister holding the Law and Justice and
Electronics and Information Technology portfolios in the Government of India.
Hence, (D) is the correct answer.
48. The 19th Shanghai Cooperation organization (SCO) Summit, 2019 was held in Kyrgyzstan from 13 to
14 June, 2019. Which of the following nations is not a founding member of SCO?
A. China
B. Russia
C. Kyrgyzstan
D. India
Explanation: The Shanghai Cooperation organization (SCO) also known as Shanghai pact is an Eurasian
Political, Economic and security alliance that was founded on 15 th June, 2001 with China, Russia,
Kyrgyzstan, Kazakhstan, Tajikistan and Uzbekistan as its founding members. The SCO succeeded the
‘Shanghai five group’ that was founded on 26 April, 1996 by all the nations that founded its Successor
(SCO) except Uzbekistan.
Currently, the SCO has 8 members with India and Pakistan officially joining it as full time members on 9
June, 2017 while previously it had 6 members which are its Founding members. The 2019 SCO summit
held in Bishkek, the capital city of Kyrgyzstan was 19 thsuch Summit since the inception of SCO.
Therefore, Except India and Pakistan, all other current members of SCO are its founding members.
49. Who has been recently named by Forbes Magazine as the youngest self-made billionaire of all time?
A. Kylie Jenner
B. Elizabeth Holmes
C. Mark Zuckerberg
D. John Collison
Explanation: At the age of just 21 Kylie Jenner is named as the youngest self-made billionaire of all times
by the Forbes magazine. She is the founder of Kylie Cosmetics Company which is booming in the U.S
Market and has a capital worth $900 million.
Option (B), Elizabeth Holmes, was the world’s youngest woman to become a self-made billionaire until
Kylie Jenner came to Picture.
Option (C), Mark Zuckerberg, is the second youngest person to become a self-made billionaire, at the
age of just 23. He is the co-founder and CEO of Facebook.
Option (D), John Collison co-founded Stripe along with his brother Patrick Collison. Stripe is an online
payments company. He entered the list of billionaires at the age of 26.
50. Recently the remains of which Indian Airforce (IAF) aircraft were found in a search operation in the
highly dense forests of Arunachal Pradesh, which went missing on 3 June, 2019 after take-off and
thereupon crashed leading to death of all the 13 air warriors on board?
A. AN-32
B. Mig-21
C. Mig-29
D. Sukhoi-Su-30
Explanation: The Russian Origin, AN-32 aircraft of the IAF went missing after 33 minutes of take-off from
Jorhat in Assam on 3rd June, 2019. Later on, after extensive search operations for several days, its
remains were found in the densely forested Mountain terrains in Arunachal Pradesh at a height of
12,000 ft. above sea level near Gatte village located very close to the Indo-China border.
There were 13 air-warriors on board at the time of crash and no one is reported to have survived. It is
the worst accident involving a military aircraft in recent years in India.
51. Which of the Following Indian city don’t have a functional metro rail network currently?
A. Lucknow
B. Jaipur
C. Hyderabad
D. Aurangabad
Correct answer: (D)
Explanation: Currently in India, there are 12 cities that have a functional Metro rail network; these
cities are Kolkata, Delhi, Chennai, Bengaluru, Hyderabad, Jaipur, Gurgaon, Mumbai, Kochi, Lucknow,
Noida, Pune and Nagpur. The Metro came to India in 1984, with Kolkata being the first Indian city to
have a Metro Rail Connectivity.
Therefore, out of the above options all have Metro rail connectivity except Aurangabad.
Explanation: DNA stands for ‘deoxyribonucleic acid’ while RNA stands for ‘ribonucleic acid’. While DNA
is a Double stranded molecule, RNA is a Single stranded molecule.
53. Which Country is planning to replace its street lights with artificial moon in one of its city by 2020?
A. U.S.A
B. Japan
C. China
D. Russia
Explanation: China has cleared its intention that it will soon replace street lights in the city of Chengdu
with an artificial moon that will orbit the Chinese city and will illuminate the city in place of costlier
street lights that eats up a lot of electric energy. The country’s aim is to replace all the street lights in its
urban areas with artificial moon in the future. The artificial moon will use a huge mirror that will reflect
the sunlight to illuminate the city and will cut the costs by $174 million per year.
Explanation: The International Court of Justice also popularly known as the world court is located in The
Hague, Netherlands, and its official address as printed on its Press Release is ‘Peace Palace,
Carnegieplein 2, 2517 KJ, The Hague, Netherlands’.
The ICJ was established in 1945 as a successor of Permanent court of International Justice (PCIJ) which
was dissolved after the Second World War.
[For Reference:
The ICJ comprises of a Total of 15 Judges and Justice Dalveer Bhandari is currently the only Indian Justice
sitting at the ICJ presently.
55. Which Indian Personality won the Ramon Magsaysay Award, 2018 which is often referred as the
Nobel Prize of Asia?
A. Sonam Wangchuk
B. Bezwada Wilson
C. Bharat Vatwani
D. Both (A) and (C)
Explanation: Sonam Wangchuk and Bharat Vatwani are the two Indians among the six recipients of
Ramon Magsaysay award in 2018 which is regarded as Asia’s highest honor and is often called as the
Asian Version of Nobel prize. The other four Recipients are:
(i) Youk Chhang from Cambodia;
(ii) Maria de Lourdes Martins Cruz of East Timor;
(iii) Vo Thi Hoang Yen of Vietnam;
(iv) Howard Dee of the Philippines
Sonam Wangchuk is an engineer and founder of Students Educational and Cultural Movement of Ladakh
(SEMCMOL) in 1988 he also played an active role in launching ‘Operation New Hope’ in 1994 in
collaboration with the Government in order to bring reforms in Government School system. He taught
numerous poor Ladakhi students who could not afford the cost of studies. His Excellency has harvested
such an optimistic result that success rate of students in Ladakh in matriculation exams got up many-fold
times from a mere 5% in 1996 to a whopping 75% in 2015.
Dr. Bharat Vatwani is a psychiatrist based in Mumbai and he had contributed exceptionally well in
rescuing mentally-ill street paupers and reuniting them with their families. He along with his wife kicked
off an informal operation of bringing mentally-ill street persons to their private clinic for treatment and
later established ‘Shraddha Rehabilitation Foundation’ in 1988 aimed at rescuing mentally-ill persons
living on the streets, and providing them free shelter, food, and psychiatric treatment and reuniting
them with their families.
Ramon Magsaysay Award was established in 1957 in the memory of third Philippine President after
whom the award is named.
Option (B), i.e. Bezwada Wilson was the Indian Recipient of Ramon Magsaysay award in 2016 and not in
2018.
A. India
B. USA
C. UAE
D. Norway
Explanation: Shaikh Khalifa Bin Zayed Al Nahyan, the president of United Arab Emirates (UAE) declared
that the West Asian Nation will mark 2019 as the Year of Tolerance. The object behind it is to highlight
the role of UAE as the global capital for Tolerance.
The ‘Year of tolerance’ in UAE kick started with the visit of ‘Pope Francis’, who is the head of Catholic
Church and Sovereign of the Vatican City. The UAE is home to nearly one million Roman Catholics, and a
majority of them hails from Philippines and India.
57. American novelist, Richard Powers has been awarded ___________ prize, in 2019 for his book ‘the
Overstory’.
Explanation: Richard Power’s epic novel ‘The Overstory’ has won him ‘The Pulitzer Prize’ for 2019 in the
Fiction category. The book was also nominated for the ‘Man booker prize’ in 2018 but finally it was won
by Anna Burns for her Novel ‘Milkman’.
The Pulitzer Prize is awarded in 21 categories every year and is presented by Columbia University in New
York. It is awarded for excellence in newspaper Journalism and other literary achievements.
58. Recently the Union Cabinet decided to extend the Reservation benefits in promotions for SCs, STs
and OBCs in which State/Union Territory?
Explanation: The Union Cabinet on 4th March, 2019 decided to extend Reservations in promotions for
Schedule Castes (SCs), Schedule tribes (STs) and other backward castes (OBCs) in the state of Jammu and
Kashmir and also to the people living along the International Borders.
59. Who has been appointed as India’s first ever Lokpal Chairperson?
Explanation: Five years after the enactment of Lokpal and Lokayukts act that received President’s assent
way back in 2014; finally in 2019 Justice Pinaki Chandra Ghose (P.C. Ghose) has been appointed as
India’s first Lokpal Chairperson.
The high-level selection committee comprising of Prime Minister, Chief Justice of India, Lok Sabha
speaker and senior advocate Mukul Rohatgi as panel’s eminent jurist member cleared the name of
Justice P.C. Ghose, following pressure from the Supreme Court due to excessive delay in the process.
[For reference:
Lokpal is set up at center and Lokayukts are meant for states to enquire allegations of corruption against
Public functionaries]
60. Congress President Rahul Gandhi is elected to the 17 th Lok Sabha from which constituency?
A. Amethi
B. Wayanad
C. Vadakara
D. Kannur
Explanation: In the 2019 Lok Sabha elections, Rahul Gandhi filed nomination from two seats, namely
Amethi and Wayanad. From Amethi in U.P he was defeated by BJP candidate, Smriti Irani, however from
Wayanad in Kerala he managed to win and was elected from there to the 17 th Lok Sabha.
Option (A), i.e. Amethi has been Rahul Gandhi’s Lok Sabha constituency for three consecutive terms, i.e.
for 15 years from 2004 to 2019 but in 2019 Smriti Irani was elected to Lok Sabha from this constituency.
Option (D), i.e. Kannur has elected K. Sudhakaran to the 17 th Lok Sabha.
61. In 2019 General elections, the ruling NDA Government clean sweeped in how many States/Union
territories?
A. 12
B. 9
C. 7
D. 10
Explanation: The BJP led National Democratic Alliance (NDA) that came back to power in the 2019 Lok
Sabha Elections with a whooping majority of 357 seats, won all the seats, i.e. clean sweeped in 10
States/Union territories.
The 10 States/Union territories are Gujarat, Rajasthan, Himachal Pradesh, Delhi, Uttarakhand, Tripura,
Arunachal Pradesh, Haryana, Chandigarh and Daman & Diu.
However, in Kerala, Andhra Pradesh, Tamil Nadu and Puducherry, BJP was unable to win even a single
seat.
A. Pradhan Mantri Ujjwala Yojna & Ministry of Petroleum and Natural Gas
B. Ujala Scheme & Ministry of Women and child development
C. Startup India Campaign & Ministry of Commerce and Industry
D. Pradhan Mantri Ujjwala Yojna & Ministry of Women and child development
Explanation: Prime Minister Narendra Modi on 1st May, 2016 launched Pradhan Mantri Ujjwala Yojna
(PMUY) with a view to distribute 50 million free LPG connections to women coming from BPL families.
The scheme was initiated by Ministry of Petroleum and Natural Gas and has till date released
7,22,40,939 LPG connections under the PMUY Scheme (as per the official records). The scheme was
allocated an amount of Rupees 800 billion and presently covers 714 districts of the country.
Option (B), i.e. ‘Ujala Scheme’ was launched in January, 2015 with the aim of distributing free LED bulbs
to BPL families and it is currently the largest LED distribution Programme in the world, with distribution
of around 30 crore LED bulbs.
Option (C) which is ‘Startup India Campaign’ was launched by Prime Minister Narendra Modi during his
Independence Day speech on 15th August, 2015, with the object to promote start-ups and small scale
industries in India, initiated by Ministry of commerce and Industry.
63. Supreme Court took Suo Motu cognizance against which State government, for its decision to alter
changes in the Punjab Land Preservation act, 1900?
A. Punjab
B. Himachal Pradesh
C. Haryana
D. Delhi
Explanation: The Supreme Court bench comprising Justice Deepak Gupta and Arun Mishra took Suo
Motu cognizance and reprimanded the Haryana Government, which seeked critical amendments in
Section 3 and Section 4 of Punjab land Preservation Act, 1900 (PLP act) in order to legitimize real estate
construction on the land protected under the act. The court said “It is really shocking, you are destroying
the forest, and it is not permissible. Do you think you are Supreme? You are not Supreme the Rule of law
is Supreme.”
The century old PLP act, 1900 is aimed to conserve subsoil water and wildlife in the Aravallis.
[For Reference:
Suo Motu cognizance means an action taken by the court on its own.]
A. Sikander e Saini
B. Ali Gurshasp
C. Zafar Khan
D. Alauddin
Explanation: Alauddin Khalji was the second and the most powerful ruler of the Khalji dynasty. He came
to throne in 1296 AD after killing his Uncle and father in law, Jalal ud din Firuz Khalji and declared
himself, the ruler of Delhi Sultanate. He was the son of Shihabuddin and was named Ali Gurshasp at the
time of his birth. Due to early death of his father he was brought-up by his uncle and father in law, Jalal
ud din Firuz Khalji. He wanted to become like Alexander, the conqueror and thus gave himself the title
‘Sikander e Saini’ which meant ‘the second Alexander’.
He was an orthodox ruler and the historians often surround him with several controversies, including
the one regarding his sexuality.
65. Recently ISRO launched its high throughput communication satellite GSAT-29 from Satish Dhawan
Space Centre in order to monitor sensitive organs along the border and provide Internet connectivity in
Remote areas. What was the name of the mission/exercise in which the GSAT-29 was launched and
released in the Orbit?
Explanation: On 14th November, 2018 the GSLV MkIII-D2, the second developmental flight
of Geosynchronous Satellite Launch Vehicle Mark III (GSLV MkIII) successfully launched GSAT-29, which
is a high throughput communication satellite aimed at monitoring sensitive organs along the border and
also tracking enemy ships in the Indian Ocean, by virtue of its ‘Geo Eye’ technology (it has a unique high-
resolution camera) and to provide internet connectivity in remote areas of J&K and north-eastern states.
It was launched from the Second Launch pad (SLP) at Satish Dhawan Space Centre at Sriharikota in
Andhra Pradesh. The GSAT-29 is 3,423 kg in weight and is the heaviest satellite launched from India till
date.
A. Albert Einstein
B. Werner Heisenberg
C. J.J. Thomson
D. Ernest Rutherford
Explanation: Ernest Rutherford, who was a New Zealand born British Physicist, is popularly known as the
Father of Nuclear Physics. He discovered the concept of ‘Radio-active half-life’ and also differentiated
between Alpha and Beta Radiations.
He was awarded Nobel Prize in Chemistry in 1908 for his meritorious work in the field of radioactive
substances and was the first Canadian Nobel laureate. The element ‘Rutherfordium’ is named after Ernst
Rutherford. Moreover, the unit of Radioactivity which is ‘Rd’ stands for ‘Rutherford’. He is regarded as
one of the greatest experimentalist of the world.
67. Who is the author of the book ‘Law, Justice and Judicial Power: Justice P.N. Bhagwati’s Approach’
that was launched on 8th February, 2019 by Chief Justice of India, Ranjan Gogoi?
A. Ashok K. Ganguly
B. Mool Chand Sharma
C. Arun Shourie
D. Ram Jethmalani
Explanation: On 8th February, 2019 the Chief Justice of India (CJI) Ranjan Gogoi launched the book ‘Law,
Justice and Judicial Power: Justice P.N. Bhagwati’s Approach’ written by Mool Chand Sharma. President,
Ram Nath Kovind received the first copy of the book from CJI on the same date and officially launched
the book.
The book contains 24 elaborate essays regarding Justice P.N Bhagwati’s landmark Juristic works and
Judicial Pronouncements. The book also refers to his contribution towards introducing the Concept of
Public Interest Litigation (PIL) in India. Justice Bhagwati is known as the father of PIL in India.
The author of the book, Professor (Dr.) Mool Chand Sharma is a renowned professor of law at the Delhi
University. He was also a full time member of law Commission of India.
Hence (B) is the correct answer.
68. On 13th June, 2019 which Country’s highest court legalized same-sex marriage?
A. Brazil
B. Kenya
C. Malaysia
D. Ecuador
Explanation: Ecuador’s highest Court’s 9 judge constitutional bench, ruled with five-to-four majority in
favor of legalizing same-sex marriage in the Catholic-Majority country. It has been a major event for
LGBTQ rights not only in the nation but in the whole world, making international news worldwide.
The judges on Quito’s (capital city of Ecuador) Constitutional court declared the prevailing legislations
relating to marriage as discriminatory on the basis of gender and thus, unconstitutional. The law prior to
this judgement recognized marriage as a union between a man and a woman only.
69. National Science Day is celebrated on which date every year and what was its theme in 2019?
A. 28th February & Science and Technology for specially abled persons.
B. 24th February & Science and Technology for a sustainable future.
C. 28th February & Science for People and People for science.
D. 31st May & Make in India: S&T driven innovations
Explanation: On 28th February every year, National Science Day is celebrated in India to embark the
discovery of ‘Raman Effect’ by great Indian Physicist ‘Sir Chandrasekhara Venkata Raman’ on the same
date (28th February) in 1928. Raman Effect is a discovery related to change in wavelength of light and ‘CV
Raman’ was awarded Nobel Prize for it in 1930, making it India’s first Nobel Prize in the field of Science.
Therefore, National Science Day is celebrated in India on 29 th February every year since 1987. The theme
for the year 2019 was ‘Science for people and people for science’.
Option (A) is half correct, because the theme stated in it is of National Science Day, 2017.
Option (B) is wrong because 24th February is marked as ‘Central Excise Day’.
Option (D) is incorrect because 31st may is recognized as ‘World No Tobacco Day’.
Explanation: On 8th March, 2019 five judge Constitutional bench of Supreme Court headed by CJI Ranjan
Gogoi, referred the Ram Janmbhoomi- Babri Masjid Dispute in Ayodhya to a 3-member mediation panel,
which was created by the court itself.
The Panel includes Spiritual Guru Sri Sri Ravi Shankar, Retired Supreme Court Justice Fakkir Mohamed
Ibrahim Kalifulla and Mediation lawyer, Senior Advocate Sriram Panchu. The panel will be headed by
Justice Fakkir Mohamed Ibrahim Kalifulla. It is interesting to note that all three members hails from the
state of Tamil Nadu.
The Proceedings of Mediation panel was held in Faizabad, in Uttar Pradesh and it was all in-camera
proceedings.
Therefore, Except Justice Ashok Bhushan, all others stated in the option were members of this
Mediation Panel.
71. Which two Indian Army Units were involved in the famous Surgical Strikes which took place on 29 th
September, 2016 over the Militant launch-pads in Pakistan, along the Line of Control?
Explanation: On 29th September, 2016 Indian Army’s Northern Command along with Parachute
Regiment (4 and 9 para Special Forces) successfully conducted a surgical strike to avenge the Uri Attack
on the Indian Army’s 4 Para (Special Forces) on 18 th September, 2016, which was one of the deadliest
attack on the security forces in Kashmir valley, leading to the death of 19 Soldiers, 4 heavily armed
Militants of Militant group Jaish-e-Mohammed were involved in the attack.
Exactly after 11 days of the attack, the Indian army avenged the death of its 19 Martyrs by killing more
than 70 militants across the LOC in POK by successfully conducting the Surgical Strikes wherein the
Forces Destroys the suspected launchpads that were involved in the Uri attack.
Hence, (A) is the correct answer.
A. Prithviraj Kapoor
B. Devika Rani
C. Devanand
D. Vinod Khanna
Explanation: Devika Rani was the first recipient of Dadasaheb Phalke Award, receiving it in 1969. She is
also known as ‘the first lady of Indian Cinema’. Her debut movie was in 1933 named ‘Karma’, which was
the first Indian Film in English language and also the first Indian film featuring an on-screen kiss.
Apart from being the first person to receive Dadasaheb Phalke Award, she had also been awarded with
Padma Shri in 1958. She and her 1st Husband Himanshu Rai jointly founded Bombay Talkies in 1934,
which was one of the leading movie studios of that era and produced a large number of Hit films with
actor Ashok kumar who was the leading actor of their studio.
Dadasaheb Phalke award is the highest award in Indian Cinema and is awarded for outstanding
contribution to the growth and development of Indian Cinema. The award is named after Dadasaheb
Phalke in his honor. He directed India’s first full length feature film, Raja Harishchandra in 1913 and is
widely known as the ‘father of Indian Cinema’.
All other Options stated above have also been awarded with this award but Devika Rani is the first
person to be its awardee.
A. Callisto
B. Ganymede
C. Titan
D. Europa
Explanation: Titan, which is the largest Moon of the planet Saturn, is the second largest moon in our
Solar System. It has a diameter of 5,150 km, and was discovered by Christiaan Huygens, a Dutch
Astronomer in 1655. Apart from earth it is the only known body in our solar system that has water
bodies. It has seas and lakes on its surface that are filled with liquid hydrocarbons. It has a dense
atmosphere similar to that of earth. Including titan, Saturn has 62 moons out of which 53 are named.
Option (B), Ganymede is the largest moon of our Solar system. It is one of the 79 moons of Jupiter, and
is 5,262 kilometers in diameter.
Explanation: On 31st May, 2019 Vice Admiral Karambir Singh was appointed as the new chief of Naval
Staff. He succeeded Admiral Sunil Lamba who retired on 30 th May, 2019.
Vice Admiral Karambir Singh is the first Helicopter Pilot to occupy the office of Navy Chief.
75. Goa Chief Minister and senior BJP leader, Manohar Parrikar who passed away recently was suffering
from which form of Cancer?
A. Lung Cancer
B. Pancreatic Cancer
C. Blood Cancer
D. Stomach Cancer
Explanation: Manohar Parrikar who served as 3 times chief Minister of Goa and was also former
Defence Minister of India passed away at the age of 63 fighting with Pancreatic Cancer on 17 th March,
2019. He was also a former Rajya Sabha member from Uttar Pradesh.
76. The 17th Edition of Varuna exercise that took place from 1-10 May, 2019 at Goa coast is conducted
between India and which nation’s navies?
A. U.S.A
B. Russia
C. France
D. Spain
77. The battle of Saragarhi, which is regarded as one of history’s greatest ‘last stand’ battles was fought
between Afghan Warriors and which regiment of the British Indian Army?
Explanation: The battle of Saragarhi was fought between 21 soldiers of 36 th Sikhs regiment of the British
Indian Army (now 4th battalion of the Sikh regiment of Indian army) and 10,000 Afghan Warriors
(Pashtun Orakzai tribesman) on 12th September, 1897.
In consequence, all the 20 soldiers along with their commander, Havildar Ishar Singh were killed, but
they fought with extreme courage, that their martyrdom was paid tribute even in the English
parliament.
The Saragarhi fort was of high strategic importance for the Britishers because it was located between
Fort Gulistan and Fort Lockhart, and acted as a communication channel between the two forts.
The 4th Battalion of the Sikh regiment commemorate 12 th September every year as ‘Saragarhi Day’.
78. On 28th May, 2019 India, Sri Lanka and which nation signed an MOU at Colombo to jointly develop
the East Container Terminal at the Colombo Port?
A. China
B. Japan
C. Maldives
D. Myanmar
Explanation: India, Sri Lanka and Japan decided to sign an MOU (Memorandum of understanding) with a
view to develop a container terminal at the port of Colombo that will facilitate Large Container ships to
enter.
The container Terminal is intended to attract major investment from China under its Belt and Road
initiative.
Sri Lanka has been one of the countries which are drawn by China under its Belt and Road Initiative that
was announced by Chinese President, Xi Jinping with the view to build infrastructure worth $1 trillion in
order to support increasing trade and economic ties and also enhance China’s interest around the globe.
China Communications Construction Company which is a state owned company is engaged in building a
huge project in the Port city of Colombo in Sri Lanka, which includes building of Malls, Marina, hospital
and numerous apartments and homes. Their vision is to develop a financial district which will act as a
new hub between Singapore and Dubai.
Japan, under its Free and Open Indo-Pacific Strategy has signed an agreement (MOU) with India and Sri
Lanka to support the already started initiative by China and in order to pose as a competitor to the
Chinese Superpower.
79. The Ease of doing business Report is released annually by which Institution/Organization?
A. World Bank
B. IMF
C. World Economic Forum
D. United Nations
Explanation: The ‘Ease of doing business’ report is released by World bank every year.
The 2019 report on Ease of doing business was released by World Bank on 31 st October, 2018. India was
ranked 77th in the report and it is a great improvement for India, that jumped 23 spots from 100 th rank
last year (2018) to 77th Rank this year (2019). India is also recognized in the report as one among the top
10 improving countries consecutively for the second year, and it is the only large country to be
recognized under the category of Top-ten improving countries.
The Report includes 190 countries and is topped by New Zealand for the third consecutive year, while
Somalia has bagged the last spot.
[For Reference:
The World Bank comprises of two institutions, namely, the International Bank for reconstruction and
development (IBRD) and International Development association (IDA). The IBRD has 189 member
countries while IDA has 173 countries as its members, presently. Headquarter of World bank is in
Washington D.C, in U.S and it was formed in July, 1945. David Robert Malpass is its current President.]
80. Who has been appointed as Pakistan’s new ISI chief?
Explanation: Lieutenant General Faiz Hameed is appointed as the new ISI chief on 17 th June, 2019. He is
the 24th director general of ISI (Inter-service Intelligence) and succeeded Asim Munir who was removed
from the post after 8 months of his appointment.
Faiz Hameed is an ex-head of ISI’s counter Intelligence wing and he is also a close companion of
Pakistani army Chief, General Qamar Bajwa. Option (A), i.e. Syed Shahid Hamid was the first ISI chief.
[For Reference:
ISI stands for Inter-Services Intelligence, and it is a premier intelligence agency of Pakistan founded in
1948 as an independent unit. Earlier it was a part of Intelligence Bureau. ]
A. Geneva
B. Paris
C. Washington
D. New York
Explanation: The United Nations Educational, scientific and Cultural organization (UNESCO) is
headquartered in the French Capital Paris. It was founded on 4 th November, 1945, and its motto is
‘Building Peace in the minds of Men and Women’.
In India presently there are 37 UNESCO World Heritage sites, after Victorian and Art Deco Ensembles of
Mumbai were added to the list in 2018.
82. Which Movie won most number of Awards in the 64 th Filmfare awards held in Mumbai?
A. Raazi
B. Andhadhun
C. Padmaavat
D. Both (A) and (B)
[For Reference:
83. Which Planet in our Solar System has the longest day?
A. Mercury
B. Earth
C. Mars
D. Venus
Explanation: Venus has the longest day in our Solar System. It has a day of 243 days and its day (that
depends upon rotation) lasts longer then its year (that depends upon Revolution). It completes its one
revolution around the sun in 224.65 days while its one rotation is completed in 243 days; hence its day is
even longer then its year.
Moreover, Sun rises from the west on Venus because it has a retrograde rotation, i.e. it rotates in
opposite direction of the rest of the planets. It is often seen with naked eyes at the time of dawn from
earth and is therefore called ‘Morning Star’.
A. Study of Punishments
B. Study of Soil
C. Study of Insects
D. Statistical study of elections.
Explanation: Pedology is the study of soil in its natural environment. It is one of the two main studies
related to soil science, the other being ‘edaphology’.
85. Who won the best director award at the 91st Academy Awards also known as Oscar Awards held at
Los Angeles on 24th February, 2019?
A. Spike Lee
B. Alfonso Cuaron
C. Damien Chazelle
D. Christopher Nolan
Explanation: Alfonso Cuaron, who is a Mexican Director, has won the award for the best director in the
2019 Oscars held at Dolby theatre in Hollywood, Los Angeles for his movie Roma. He won this award for
the second time in his career. For the first time he won the award for best director in 2013 for the movie
‘Gravity’.
[For reference:
Rami Malek won the award for best Actor for his movie ‘Bohemian Rhaspdy’;
Olivia Colman won the award for best actress for her movie ‘the favourite’]
86. Which out of the following Indian Projects found Merit at 2018 World Architecture Festival Awards?
A. The Street
B. Thiruvalluvar
C. The Pearl Academy of Fashion
D. Both (A) and (B)
The Pearl Academy located in Jaipur was the winner in completed buildings category in 2009.
Explanation: On 19th January, 2018 Mr. Shashi Tharoor who is an Indian Politician and senior leader of
Indian National Congress, as well as a Writer and diplomat launched his book titled ‘Why I am a Hindu’.
The 320 page book revolves around the History of Hinduism, and about how the religion evolved and is
still evolving in accordance to its surrounding socio-cultural environment. The book also talks about the
so called ‘Hindutva Movement’ in the present scenario.
88. Who is the author of the book titled ‘Triple Talaq: Examining Faith?
A. Salman Khurshid
B. Noor Zaheer
C. Salman Rushdie
D. Ravi Shankar Prasad
Explanation: Indian Politician, Senior designated Advocate, law teacher and a writer, Salman Khurshid
authored the book titled ‘Triple Talaq: Examining faith’. The book was launched in 2018 following the
Supreme Court’s verdict on Triple Talaq in ‘Shayra Bano case’ questioning the constitutional validity of
‘Instant Triple Talaq’ and declaring the practice to be unconstitutional. The book has a mediocre mass of
230 pages and a huge citation of various landmark cases has been done in it, including a detailed
discussion of the Shayra Bano Judgement. The book was originally published by ‘Oxford University
Press’.
Explanation: Lee Chong Wei is a Malaysian Badminton star. He had won three Olympic Silver medals in
Badminton for Malaysia in 2008, 2012 and 2016; in addition he was ranked world no. 1 for 349 weeks
(including a 199 week unbreakable streak). He also won various medals at Asian Championships,
commonwealth games, Asian Games, World Championship etc.
He is diagnosed with an early stage nose cancer, which is a major reason for his decision to declare
retirement from his grandeur sports career.
90. Who is the only Indian Sportsperson to be featured in the Forbes list of world’s highest-paid athletes
in 2019?
A. M.S Dhoni
B. Virat kohli
C. Saina Nehwal
D. Sania Mirza
Explanation: Forbes 2019 list of ‘the world’s 100 highest-paid athletes’ was released on 11 June, 2019.
Indian Cricket captain Virat Kohli is the sole Indian entry in the list. He has been named for the second
consecutive year in the list, however despite his income been increased by $1 million, his ranking in the
list has slipped from 83rd spot to the last spot (100th rank). Kohli’s total earning for the last 12 months
has been $25 million.
[For Reference:
The list is topped by Barcelona and Argentina Football captain, Lionel Messi whose total earning
amounts $127 million. Messi replaced retired boxer, Floyd Mayweather who was at Rank 1 in 2018 and
became the World’s top-paid athlete. He is followed by his arch-rival and Portuguese Football superstar,
Cristiano Ronaldo, who with earnings of $109 million is on the 2 nd Spot.
Serena Williams is the only Female sportsperson in the world to be named in this list.]
Directions: This section consists of fifty (50) questions. Each question consists of legal principle(s)
(hereinafter referred to as ‘principle‘) and facts. Such proposition may or may not be true in the real and
legal sense, yet you have to conclusively assume them to be true for the purposes of this section.
Principles have to be applied to the given facts to arrive at the most reasonable conclusion. Only one of
the alternatives, i.e., (A), (B), (C), or (D) is the most reasonable conclusion. In other words, in answering
the following questions, you must not rely on any principle except the principles that are given herein
below for every question. Further you must not assume any facts other than those stated in the
question. The objective of this section is to test your ability in legal aptitude, study of law, research
aptitude and problem solving ability even if the ‘most reasonable conclusion’ arrived at may be absurd
or unacceptable for any other reason.
112. Principle: Agreements, the meaning of which is not certain, or capable of being made certain, are
void.
Facts: Rohit agrees to sell one of his bats to Dhawan for an amount decided by kohli.
A. The meaning of the contract is certain, and hence it is valid.
B. The agreement between the two is not a valid Contract because a third party (Kohli) had
interfered in it.
C. The meaning of the agreement is uncertain, making it Void.
D. There is no Contract as no written agreement is signed.
Correct Answer: (C)
Explanation: Section 29 of the ICA, 1872 states that ‘Agreements are void if they have an uncertain
meaning’. The above fact indicates that Rohit had more than one bat and he agreed to sell any one out
of them. Therefore, no Specification as to which bat was to be sold was laid down in the agreement,
making its meaning uncertain. The agreement is Void because there is uncertainty with regard to the
subject matter (Bat) of agreement.
Option (B) gives a wrong reason for making the agreement void (invalid contract) because a contract
may have more than two parties. Moreover, Kohli is not even a party to the contract and his
interference (deciding of amount) is legal because it is made with the ‘free consent’ of both the parties
to the agreement.
Option (D) is an incorrect statement because a Contract can be either ‘Written’ or ‘Verbal’. Section 4 of
the ICA, 1872 says that communication of a contract can be made either ‘orally’ or in ‘written’.
Hence, (C) is the correct answer.
113. Principle: The Communication of a Proposal is complete when it comes to the knowledge of the
person to whom it is made.
Facts: Amit and Narendra were close childhood friends. When Narendra returned from America after
pursuing his studies he decided to start a small business along with Amit, who was not well-educated
but son of a big landlord, having a lot of Money. In pursuance, Narendra sent a mail to Amit offering him
to join the business.
Decide:
A. Communication is not yet completed, and it will be completed only when Narendra meets Amit
personally and propose him to join
B. The communication of Proposal/offer got completed the moment when Narendra Sent the mail
offering Amit to join
C. The communication of Proposal will complete once, Amit receives and reads the mail
D. The Mail sent by Narendra to Amit was not an offer but an invitation to offer.
Correct Answer: (C)
Explanation: The above given Principle resembles with Section 4 of the ICA, 1872 which states the
condition when a communication of Proposal gets completed. As per the Principle, Communication of a
Proposal/Offer is completed when the person to whom it is made gets to know about it. Therefore,
Option (C) is the correct answer because Amit will get hold of the knowledge of that offer only, the
moment when he will read the mail.
Option (A) is incorrect because the bare requirement for completion of a communication of proposal is
that, the other party (to whom proposal is made) should get to know about it, and it could be either
through a personal meeting (Direct) or via any other mode of Communication (Indirect).
Option (B) is completely wrong because it doesn’t match the requirements as laid down in the Principle.
Option (D) is absurd because ‘an invitation to offer’ is a completely different thing, which is in no way
relatable to the facts of the present question. An ‘invitation to offer’ means, inviting others to
offer/propose, with regard to which, that person has complete autonomy to either accept or reject it.
For e.g. An Auction is an invitation to offer, or a Product placed in a shop is an invitation to offer, which
is made to the buyer of that product by the seller.
Hence, (C) is the correct answer.
115. Principle: Acceptance of Offer/Proposal should not come back with a Counter offer.
Facts: Raj went to Simran and offered his second-hand mobile phone to her for an amount of Rupees
20,000. In response to the offer, Simran showed her willingness to purchase the Mobile for Rs 15,000.
A. Since, Simran had accepted the offer made by raj, they have entered into an Agreement
B. Simran has not accepted the Proposal of Raj
C. Raj Didn’t made any offer to Simran and it was just a suggestion
D. It is not clear if Simran has accepted the proposal of Raj
Correct Answer: (B)
Explanation: According to the above stated principle, an offer made by a promisor should be accepted
by the Promisee as it is, without making any counter condition or offer. In other words it can be said that
‘Acceptance of Proposal must be the exact mirror image of the proposal’. Therefore, if the Promisee (the
person to whom offer is made) is accepting the offer, he/she must revert back with a simple affirmation
without attaching any additional conditions (counter offer) to it.
In the above facts, Raj offered Simran his mobile phone for rupees 20,000 but in response, Simran,
showing her willingness to Purchase it for rupees 15,000 counter-offered Raj. Since, she didn’t accept
the offer made by Raj as it is, acceptance of that offer was not made.
Option (C) is irrelevant as the conditions of an ‘offer/proposal’ as stated in section 2(a) of the ICA, 1872
are fulfilled.
Option (D) is also a misguiding statement because it can be clearly inferred without any reasonable
doubt that acceptance is not made.
116. Principle: Possession and Ownership are two different sides of a same coin. While the former is
Physical the latter is Legal.
Facts: Ram and Rajesh were Business Partners and Close friends. Rajesh often used to visit outstations
for attending meetings regarding their business, while Ram used to stay at their Office headquarter only.
For travelling purposes Rajesh used a car that was registered in Ram’s name. One day while Rajesh was
driving that car he killed a person who was trying to cross the road. When sued by him in the court,
Rajesh pleaded that he was not the owner of that car.
A. Rajesh should be Prosecuted and Punished for the Heinous crime he committed under section
304 A of IPC, 1860
B. Both Ram and Rajesh should be held for the offence
C. Only Rajesh should be held guilty
D. Rajesh is the possessor, while ram is the Owner.
Correct Answer: (D)
Explanation: The principle stated above is derived from the provisions of Transfer of Property Act, 1882.
According to it, Possession means physical presence of a property under one’s control, whereas
Ownership means a legal right of a person over a property at the exclusion of all others, but it doesn’t
necessarily includes Physical presence. For e.g. A landlord is the owner of a house, while a tenant is a
Possessor. A person who has acquired a property through illegal means can also be a possessor but
never the owner of that property. For instance a Thieve, can Possess a property which he stole but he
can’t own it.
Rajesh was driving the car and it was under his control, therefore the car was in his possession, whereas
Ram was the legal owner of the car.
Option (A) is True but irrelevant in the context of the Principle.
Option (B) also relates to Criminal law.
Option (C) is also incorrect on the same footing.
Hence, (D) is the correct answer.
118. Principle: A person usually with unsound mind can also enter into a contract when under sound
state of mind.
Facts: Shyam is suffering from a mental disorder, in which he starts thinking himself to be a Lion. Most
of the time of the year he is found in this mental ailment, but during the month of January and February,
he becomes completely normal.
Can he enter into a Contract?
A. He can’t make a Contract.
B. He can make a Contract anytime he wishes.
C. He can make a Contract but only in the month of January and February.
D. He can make a Contract only with a person of unsound mind.
Correct Answer: (C)
Explanation: According to the Principle which is taken from Section 12 of Indian Contract Act, sound
state of mind of a person is required at the time of making or entering into a contract and not always. It
means that even a person of unsound mind can enter into a contract when his mind is normal and
similarly if a normal person sometimes becomes lunatic then he can’t enter into a contract during that
phase.
Hence, Shyam may enter into Contract or make a valid Contract when his mental state is normal (during
the month of January and February), but not when he is under mental ailment.
Option (A) is not absolutely true, and depends upon his mental condition.
Option (B) is false, because his wish doesn’t matters, his mental state does.
Option (D) is an absolutely wrong statement, completely opposite to the above principle.
121. Principle: When consent of a person is obtained through Coercion then the Contract is enforceable
from the side of that party who dominated the will of other party by Coercion, while the coerced party
has the freedom to either enforce or declare it void.
Facts: Z is a big Builder of the city of Delhi. He wanted to build a mall on the land of X, but X was
unwilling to sell his land to Z.
Z tried all possible ways to convince X, he also offered him twice the money than the actual market
price, but X didn’t agreed to sale off his land at any cost. Finally, Z decided to use his Muscle Power and
took X’s signature on the property papers by keeping X on Gun point.
A. Consent of X is obtained through Coercion; hence it is voidable at the option of X.
B. Consent of X is obtained through Coercion; hence it is voidable at the option of Z.
C. The contract is void ab initio
D. The Contract is voidable at the option of both parties.
Correct answer: (A)
Explanation: Z obtained X’s Consent by threatening him of dire consequences. Thus, the consent of X is
not free and is obtained through coercion. On applying the above Principle to the facts it can be inferred
that the Contract may be declared void by X or may be enforced if X wants, hence it is voidable at the
Option of X, whereas on Part of Z it is absolutely enforceable and he has no power ( like X) to declare it
void.
Option (B) is wrong because it gives an assertion opposite to the principle.
Option (C) is false because section 19 of Indian Contract act makes a contract voidable in which consent
is obtained through fraud.
Option (D) is false and not relevant according to the principle because the contract should be voidable at
the option of only X because he is the only one, whose Consent is obtained through Coercion.
Hence, (A) is the correct answer.
122. Principle: An Agreement made without consideration is Void unless expressed in writing and
Registered under the concerned law for the time being.
Facts: Mr. A was a keen supporter of Indian Cricket team. In the world cup final match of India vs
Pakistan that was a breath taking encounter between the two teams, India required a last ball six to win
the match and Dhoni was on strike. Among the spectators a boy was praying to god that Dhoni hit a
huge six outside the stadium and A was listening to his prayers. Finally, Dhoni hit a huge Maximum and
India won the Match. A became so happy and excited that he promised that boy to pay him 50,000
Rupees, but later on refused to pay.
A. It was a valid contract and A is liable to pay
B. It was a void Contract because there was no consideration.
C. It was a valid contract because consideration is present which was dhoni’s six
D. The other party to the Contract is Dhoni and not that boy.
Correct Answer: (B)
Explanation: The agreement in the above case is void because it is without any consideration and
moreover, was also not expressed in writing nor registered under any law. Section 2(d) of Indian
Contract act defines consideration and Section 25 of the act says that agreements without consideration
are void except in some special cases. In the present facts, Mr. A promised to pay that boy 50,000
Rupees out of Excitement and he didn’t expect anything from him in return (Consideration). Similar was
the situation from boy’s side too. Hence, consideration is absent and therefore the agreement is void.
Option (A) is false because Consideration which is missing is an essential of a valid contract.
Option (C) is false because Dhoni’s six can never be a consideration because the promise was made by A
after Dhoni hit the Six.
Option (D) is wrong because no proposal was made to dhoni and making of Proposal is the foremost
requirement for creation of a Contract.
Hence, (B) is the correct answer.
123. Principle: Defamation is an injury on a person’s Reputation. The defamatory statement must be
published in order to constitute the offence of Defamation.
Facts: Rahul was a great and reputed Politician in his country and everyone respected him. In addition of
being a politician, he was a social worker and a mass leader too. A Newspaper, named ‘The Express’
published an Article that alleged ‘Rahul’ to be involved in a Corruption Scam, in which he later got
convicted by the court, after the completion of his term in prison, when he came out, he observed that
people started hating him. He realized that everything that happened with him was due to that Article
Published in ‘the Express’ newspaper and in order to take avenge he sued it for Defamation.
A. Rahul will succeed in getting justice, because the Newspaper had defamed him and ruined his
life.
B. Rahul will not succeed because whatever the newspaper published was true.
C. The Newspaper never Published anything defamatory to Rahul and it was the court verdict that
defamed him
D. Rahul can’t succeed as it is the Moral duty of the media to take the truth out in front of public
Explanation: The offence of Defamation is defined in section 499 of the IPC, 1860 and the principle is a
small extract from the same. The IPC defines the offence of Defamation as ‘when a person’s reputation
is degraded in the minds of right thinking people of the society, either by spoken words or written.
Publication of a statement is a necessary element to constitute this offence. Publication simply means
when the statement comes to the knowledge of a third person (not being the accused and the victim).
However, as per the above facts, Rahul will not ‘succeed’ in suing the newspaper because ‘Justification
of Truth’ is a complete defence (Exception) to the offence of defamation and whatever the newspaper
published was proved to be true by the Court verdict. Therefore, ‘The Express’ newspaper is eligible for
the defence of truth and will escape the criminal liability under section 499.
Option (A) is inconsistent with the Principle and the definition of Defamation under section 499 of IPC.
Option (D) doesn’t states a valid reason in support of the correct answer.
124. Principle: Ignorantia facti excusat, Ignoratia legis neminem excusat: Ignorance of Fact is excusable
but Ignorance of law is not excusable.
Facts: Ghanshyam, under a misbelief during dark night hours, thought his son who was secretly entering
the house to be a Ghost and killed him.
Explanation: Ghanshyam will not be charged for any offence and he can successfully claim the defence
of ‘Mistake of fact’ under section 76 of IPC, 1860. It is to be noted that killing a Human is a crime but
killing a Ghost is not a Crime under any law (including IPC, 1860). Misjudging a person in good (Bonafide)
faith to be a Ghost or an animal is Mistake of Fact and not of law, and as clearly laid down in the above
principle that mistake/Ignorance of Fact is excusable, i.e. it is a defence to escape criminal liability but
Mistake/Ignorance of law is not. Therefore, Ghanshyam will be protected under this defence in order to
escape Criminal liability.
Option (A) is wrong because the essentials of Murder stated in Clause 1 to 4 of Section 300 are not
fulfilled.
Option (B) is incorrect because ‘Mistake of fact’ is a general defence under Section 76, Chapter IV of
IPC,1860 and according to Section 6 of IPC, 1860, culpability under any provision of IPC firstly had to be
subjected to the provisions of Chapter IV (General Exceptions).
Option (D) is absurd because Ghanshyam’s son was not a trespasser. A trespasser is a person who
illegally enters premises under lawful control of another person, but as per the above facts Ghanshyam’s
son was legally entitled to enter the house of his father and thus, he was not a trespasser. Moreover,
Section 103 of IPC, 1860 lays down the four situations wherein Right of Private defence of property may
extend to causing death, but none can be confirmed to the factual situation of the above case.
Facts: Singham is a well-reputed and dead honest Police officer posted in the city of Ghaziabad. In his
City a dreaded Criminal named Jaykant who was serving life imprisonment for brutally killing five people
of a family, including a 5 years old child just came out on parole for a term of 10 days. Jaykant eventually
went to singham’s House with his men armed with guns and other lethal weapons and tried to bribe him
for which he refused. Due to this Jaykant felt insulted and in a heat of passion pointed his loaded gun
towards Singham’s 3 year old son. In order to save his child, Singham took out his Service Revolver and
shot dead Jaykant.
A. Singham had killed a dreaded criminal who was a threat for the whole society and instead of
punishing him the Government should Honor him with Gallantry award.
B. Singham was a Police officer and, therefore he was at liberty to kill any person breaking the
law.
C. Singham can’t be punished because it was an Excusable Homicide.
D. Singham can’t be punished because it was a Justified Homicide.
Explanation: Killing of Jaykant by Singham was a justified act on his part as it was in exercise of his Right
of Private defence of body, and the threat anticipated was proportionate to the harm caused. Therefore,
section 100 of IPC, 1860 which lists down the seven situations wherein a person can even cause death of
the aggressor in exercise of his right of Private defence can be applied to this case. ‘Justified’ literally
means ‘to be a good reason or ground for doing something’ and here, Singham had a good reason to kill
Jaykant, in other words his killing was rationalized. Thus, Singham was justified in his act.
Option (A) gives a wrong reason because a criminal also has some Human and legal rights including Right
to life laid down under Article 21 of the Constitution of India, and that’s why killing or harming anyone
(including a criminal) illegally is an offence punishable under law. The Background/Profile of a victim is
immaterial in adjudging the culpability of the accused.
Option (B) is completely wrong, because Police is state machinery and it had to operate within the ‘Rule
of law’ and it can’t act arbitrarily. The work of a Police officer is to maintain law and order and to catch
the offenders and Produce them in law courts, and not to kill them.
Option (C) is not correct because the act of Singham was not excusable. Excusable Homicide includes
those in which a person is unable to understand the consequences of his/her act due to lack of mental
capability. ‘Excusable’ literally means ‘Forgivable’. Unlike Justifiable Homicides, in excusable homicides a
person is unaware about the consequences of his act and that leads to absence of Mens rea (Guilty
Intention). Some example of Excusable Homicide are: killing by a Child (below 7 years of age), Homicide
committed by a person who is of Unsound mind at the time of commission, etc. In all these cases the act
is excused for want of ‘Mens Rea’.
The basic difference between excusable and Justifiable Homicide is that: in ‘Excusable Homicide’ the
Victim is an innocent person and so is the accused/aggressor, whereas in ‘Justifiable Homicide’ the
victim is always the aggressor and the accused, an innocent person.
126. Principle: Whoever takes or entices any minor under sixteen years of age if a male, or under
eighteen years of age if a female, or any person of unsound mind, out of the keeping of the lawful
guardian of such minor or person of unsound mind, without the consent of such guardian, is said to
kidnap such minor or person from lawful guardianship.
Facts: Nupur and Rajesh loved each other and were planning to marry as soon as Rajesh completed his
Studies and got a good job in hand. Ironically, their families had a long history of enmity with each other
and were sworn enemies, thirsty of each other’s blood. However, their family members were unaware
of their love relationship. Nupur was still a Minor and her age was 16 years, while Rajesh just entered in
his 20s. Date of birth of Nupur was 10/10/2000.
Finally, Rajesh got a job as a Doctor in a big Hospital in the city of Mumbai, and resultantly the two
decided to run away and marry. On the night of 27 th September, 2018 Rajesh and Nupur escaped from
their home and married the next day.
Three days after the marriage, father of Nupur lodged a named FIR against Rajesh for kidnapping his
daughter. Subsequently, the Police held search operations and caught Rajesh and Nupur. Rajesh was
prosecuted for the charges of Kidnapping.
Decide:
A. Rajesh has committed no crime and instead Nupur’s Father should be prosecuted for harassing
the couple.
B. Both Rajesh and Nupur loved each other and the marriage tie was a consensual act between
the two and not out of Coercion.
C. Rajesh can’t be prosecuted for kidnapping because he didn’t forced Nupur to run away and
whatever she did was done by her willfully.
D. Rajesh will be booked for Kidnapping from lawful guardianship, under the IPC.
Explanation: The above stated facts fulfill all the requirements, given in the principle in order to
constitute the offence of Kidnapping from lawful guardianship as given in Section 361 of IPC, 1860.
The Principle/Definition simply states that if any person takes or entices (meaning, attracts or tempts)
any minor to get out of the custody of his/her lawful guardian and come with him, without taking
permission or consent of his/her Guardian commits the offence of Kidnapping from lawful Guardianship.
It is to be noted that consent of the minor is immaterial and that the consent of his/her legal guardian (a
father is obviously a legal guardian, under normal circumstances) matters. Since, in the above case
Rajesh took Nupur (who was still a minor) out of the custody of her lawful Guardian (her father) without
his consent, thus all essentials of Kidnapping are fulfilled and he should be booked for Kidnapping Nupur
from lawful Guardianship of his father.
Option (A) is an irrelevant Assertion and doesn’t have any connection with the Principle stated above.
Option (B) is true, but according to the Principle it is the Consent of Guardian that matters in case of
Minors.
Option (C) is wrong because physical force or Coercion is not required to be proved for constituting this
offence and the mere fact that the person (Rajesh) took the minor (Nupur) out of the keeping of her
Guardian without his consent or will makes him liable under Section 361 of IPC, even if the Minor does it
willfully or had a consent to escape with him.
127. Principle 1: Death Penalty can be awarded only in rarest of rare cases.
Facts: Raju was a prisoner who was serving life imprisonment. Next month, there was his Daughter’s
Marriage and due to his good behavior in jail he was allowed a parole of 3 weeks, so that he could
participate in his daughter’s Marriage procession. The moment he got out of the jail, he saw his enemy,
Vikas waiting for him outside. On watching him he got filled with rage, but subsequently controlled his
emotions and ignored him, but Vikas followed him and started abusing him and his Daughter.
Suddenly, Raju got into a heat of passion and attacked Vikas with a Brick on his head leading to his
instant death.
Raju was prosecuted under Section 303 of IPC, 1860 for murdering Vikas and was awarded Death
Penalty. Decide:
A. Death Penalty can’t be awarded because it was not a rarest of rare case and his Prosecution
under Section 303 is unconstitutional.
B. Death Penalty is rightly awarded to Raju, because Section 303 doesn’t have any other
Punishment.
C. Raju can’t be prosecuted for Murdering Vikas because his act came within the ambit of Right of
private defence, which is a general exception to escape penal liability under IPC.
D. Raju was justified in killing Vikas, who was himself a heinous criminal.
Explanation: Principle 1 states that death Penalty can only be awarded in those cases wherein the
manner of killing or committing the crime is rarest. The word ‘Rarest’ literally means ‘Exceptional ‘or ‘a
thing that doesn’t happen usually’. In the legal framework and in the context of the Principle, the word
‘Rarest’ means ‘a manner (of committing crime) which is so crude and barbaric that a genuine Human
mind can’t even imagine, a situation in which a Human becomes even worse than a Wild Animal’.
Therefore, in order to award death penalty, the two essential conditions required to be fulfilled are:
(i) A crime which has death Penalty as one of the Punishment should be committed; and
(ii) Such Crime should be committed in a highly rare manner/way, i.e. in such a manner in which it is not
usually committed, breaking all standards of Humanity. For instance a Cold blooded Murder of an
innocent Infant.
The Principle of rarest of rare case was propounded by the Supreme Court in Machi Singh vs State of
Punjab (1983).
However, it can be noted that in the present facts, Raju killed Vikas in a sudden heat of Passion and in a
manner which under no circumstances can be adjudged as rarest of rare (as killing someone with a Brick
is not unusual or beyond imagination).
Moreover, Section 303 of IPC, 1860 states that if a person, being under sentence of imprisonment for
life, commits murder, shall be punished with death. But it is to be noted that Section 303 of IPC can’t be
applied to the above case because despite the fact that the offender (Raju) was already serving the
sentence of Life imprisonment, his act of killing Vikas was not Murder, but culpable homicide not
amounting to murder. The Definition of Murder given under section 300 of IPC also lists some of its
exceptions, and the presence of any of the exception in the facts of the case, will take out the offence
from the purview of section 300 and make it an offence defined under Section 299 of IPC (Culpable
Homicide not amounting to Murder). Thus, Section 303 of IPC doesn’t apply on Raju.
Furthermore, Principle 3 states that any provision carrying mandatory death penalty is Ultra vires to the
Constitution, i.e. constitutionally invalid. Since, Constitution is the supreme law of the land, all laws
should run in accordance to it, and that’s why Section 303 of IPC, 1860 is Constitutionally invalid and
hence a Void provision.
Option (B) is false because the essentials to be prosecuted under section 303 are not fulfilled.
Option (C) is a false statement because, the Right of Private defence should always be proportionate to
the threat anticipated, and in the present case it was not, as Vikas was only abusing him and was not
creating any sort of threat over his life or limb.
Facts: A entered into a Contract with B to Purchase his dog. While paying the agreed amount for the dog
to B, A said “If you remain silent, then I will conclude that this dog is a pure German shepherd as
demanded by me”. Later on, when A took that Dog to a Veterinary doctor for regular vaccination, the
doctor told him that the Dog was a Mongrel (Mixed breed) and not a Pure German Shepherd.
A. B is not liable because he never explicitly said that the Dog was a pure breed.
B. B is not liable because it was A’s duty to inspect everything before entering into the contract.
D. B is not liable because he, in good faith believed that the dog was a pure German Shephard.
Explanation: Section 17 of Indian Contract Act, 1872 defines Fraud, and it mentions in its explanation
that in a situation wherein a person has the duty to speak, there silence as to material facts in order to
conceal them and to gain consent of the other party amounts to a fraudulent speech. A contract in
which consent is obtained through fraud is voidable at the behest of the party whose consent is thus
obtained, which in the present case is A.
In the present facts, A explicitly told B that his silence will mean that the dog is pure breed and ‘B’ still
remained silent which meant that he impliedly accepted what A said and his silence approved of what A
believed, thus making his silence equivalent to speech and when it was later proved to be false, it made
B liable to A for breach of contract.
Here A’s words to B created a duty over B to speak and thus his silence converted into speech itself.
Option (A) is wrong because he didn’t said explicitly but accepted implicitly that the dog was a pure
breed.
Option (B) is wrong and untreatable to the Principle. Moreover, it is an era of Caveat Venditor, i.e. ‘let
the Seller’ beware, and not of Caveat Emptor, which means ‘let the buyer beware’.
Option (D) might be true, but the facts are inadequate to prove it beyond any reasonable doubt.
129. Principle: Being a Part of Unlawful assembly is in itself a crime under Section 141 of IPC.
Facts: During the British Regime in India in 1928, the freedom fighters namely Shaheed Bhagat Singh,
Chandrasekhar Azad, Rajguru and Sukhdev gathered together to plan the murder of the British police
Superintendent, John Saunders who was responsible for the death of Lala lajpat Rai, in order to take
avenge for his death. The plan was executed only by Bhagat Singh and Rajguru and the rest of the two
were not physically involved in killing him.
Decide:
A. It was an unlawful assembly because the object of all the four members was same.
B. It was an unlawful assembly and all of them should be prosecuted under section 141.
D. It was an unlawful assembly but only two members who executed the crime were part of it, and
the rest two can’t be prosecuted under any law.
Explanation: Section 141 of IPC, 1860 defines unlawful assembly as ‘when five or more persons
compose an assembly in order to prepare and commit a Criminal act with an object common to all’. But
in the present facts there are only four people as part of the assembly, hence it can’t be said to be an
unlawful assembly.
Instead of Section 141, all of them should be prosecuted for Criminal conspiracy defined in section 120A
of IPC, 1860, which is applicable to an assembly of 2 or more persons with a common criminal object.
Option (A) is false because despite the fact that all four were acting in furtherance of common object
that was unlawful too but the no. of people should be at least five in order to create an unlawful
assembly.
Option (B) is wrong as it doesn’t validates with the definition of unlawful assembly given in section 141
of IPC, 1860 and instead, all four of them should be charged under section 120A of IPC for Criminal
Conspiracy.
Option (D) is completely false and absurd statement.
Facts: A, B, C, D and E were planning to commit dacoity in XYZ bank on next Monday, and for that they
started preparation, which included Planning and acquiring necessary things like Guns and car in order
to Commit the Crime, but the Police got an advance tip and arrested all of them even before the attempt
was made by them.
A. All of them will be punished because ‘Preparation to commit dacoity’ is Punishable under IPC.
B. Only A will be punished because he was the Mastermind and the gang leader.
C. None of them should be punished because it was just a Preparation stage.
D. Police had no right to arrest them at that moment.
Explanation: On applying the above principle to the above fact, it can be clearly inferred that no person
from the group can be punished because they have not yet attempted the crime and mere Preparation
doesn’t invite Penal liability.
Keeping aside some exceptions, preparation of all other crimes is not punishable under IPC.
Option (A) is undoubtedly correct, but doesn’t conform to the Principle. Making Preparation to commit
dacoity is Punishable under Section 399 of IPC, 1860 but in such question we had to solely apply the
Principle to the factual situation and should not use any other extra-knowledge (even if the Principle is
false).
Option (B) is an absurd statement.
Option (D) is also incorrect because Police is given vast rights under Section 41 and 42 of Criminal
procedure Code. 1973, including the one exercised above.
131. Principle: Injuria, i.e. Legal Injury is an essential ingredient for a Tortious liability to arise.
Fact: Rohan and Sohan were partners in a shop dealing in Computer software. One day, a conflict arose
between them, and resultantly Sohan left the business and later on started his own business of same
kind in front of Sohan’s Shop. As Sohan came as a Competitor to Rohan’s already well established
business, he suffered huge financial loses because of him. Consequently, Rohan sued Sohan for recovery
of damages.
Explanation: ‘Injuria’, i.e. ‘legal injury’ means violation of a legal right by an act or omission of another
person. In the present case Rohan will not succeed in bringing a suit against Sohan because he only
suffered ‘Damage’ , i.e. loss (financial loss) but that was a result of Sohan, exercising his legal right to
Practice and Propagate any trade, business or profession and didn’t resulted in violation of any legal
right of Rohan.
(i) Injuria sine Damnum, which means legal injury without any damage; and
(ii) Damnum sine Injuria, which means damage without a legal injury.
The First one is a valid ground for a tortious liability to arise but the second one doesn’t leads to tortious
liability of a person.
The above fact comes under the purview of Damnum sine Injuria and that’s why due to absence of
‘Injuria’ Rohan will be unsuccessful in bringing suit against Sohan.
Option (C) is also incorrect because opening up a lawful business in any part of the country is a legal
right and exercise of a legal right can never be unlawful.
132. Principle: A person is liable for the act of his servant done in the course of Employment.
Fact: A is the Master of B. One day while B was coming back after purchasing some vegetables, he kicked
a stone that accidently hit a car and broke its Glass. The owner of the car sued A as well as B for
Recovery of damages.
Explanation: The Principle of Vicarious liability is based upon the maxim: ‘Qui facit per alium facit per
se’ which means ‘He who does an act through another is deemed in law to do it himself.’ However, the
essential requirement for a Master’s liability to arise for the acts of his servant is that the act must be
done in the course of employment and should be somehow related to it. But, in the above facts the act
of B is an independent act, not related to the course of his employment, that’s why no vicarious liability
of A arises. Kicking of Stone was not an act done in the course of Employment and was an independent
act of Servant (B), which was not authorized by the Master (A) making B liable independently.
Option (A) is incorrect because despite the fact that there is Master-Servant relationship, the act that led
to damage was not related to the course of employment.
Option (D) is absurd as the law of Tort has evolved to deal with trivial issues only.
133. Principle: A ‘bailment’ is the delivery of goods by one person to another for some purpose, upon a
contract that they shall, when the purpose is accomplished, be returned or otherwise disposed of
according to the directions of the person delivering them. The person delivering the goods is called the
‘bailor’. The person to whom they are delivered is called the ‘Bailee’.
Facts: Jai and Veeru were part of a same gang of Dacoits. Jai was assigned a task by the Gang leader,
Gabbar to kill Thakur. In order to kill Thakur Jai procured a Gun from Veeru, under a written contract
that once he will accomplish the purpose he will return the Gun back to Veeru.
After killing Thakur, Jai absconded for several days and in order to eliminate all the evidences against
him, he threw the gun in a river.
A. Veeru is Bailee and Jai is Bailor.
B. Veeru is Bailor and Jai is Bailee.
C. It is an illegal agreement, hence a void Contract.
D. Thakur is Bailor and Jai is Bailee.
Correct Answer: (B)
Explanation: Since, the object of agreement is illegal that’s why it is a void contract. Hence Option (C)
seems to be correct but it is not relatable to the Principle. On applying the above principle to the facts,
the correct statement that can be inferred is Option (B). Since, Veeru delivered his Gun to Jai thus he is
Bailor and Jai is the Bailee because he is the person to who the Gun is delivered. So, a simple application
of the above principle to the facts concludes Option (B) to be correct, no matter that the object of the
agreement was unlawful because nothing is mentioned in regard to illegality of object in the Principle.
Option (A) is false according to the Principle.
Option (C) is true in general Sense and also correct but can’t be connected with the above stated
Principle.
Option (D) is absurd because Thakur didn’t deliver the Gun (good), which is an essential to be a bailor.
Hence, (B) is the correct answer.
134. Principle: Wrongs involving Volenti Non Fit Injuria can’t be actionable torts.
Facts: Rajan was an unemployed youth and was in search of a job of any kind, as he was not well
literate. Mr. X was owner of a Circus and was in need of a trainer for training the lions used in his Circus.
When Rajan saw the advertisement regarding vacant post for Animal trainer at Mr. X’s circus he
immediately contacted him and expressed his candidature for the above post.
Mr. X told Rajan that the Job was too risky in nature and involved threat to life and limb, but Rajan
agreed to face any degree of Risk and assured Mr. X that he was an idle choice for that job because he
had a prior experience in handling wild animals. Consequently, Rajan was hired for the job.
One day while trying to control a lion in the circus, Rajan was attacked and killed by that lion. Rajan’s
family members sued Mr. X for recovery of damages.
A. Mr. X is vicariously liable for the wrong committed by lion.
B. Mr. X is liable because he was the employer and it was his duty to ensure safe working
conditions at workplace for employees.
C. Mr. X is not liable because the lion was an independent agent.
D. Mr. X is not liable because Rajan himself voluntarily consented to face the Risk of death.
Correct answer: (D)
Explanation: Volenti Non Fit injuria means ‘to a willing person, injury is not done.’ In simple words it
means that if a person has voluntarily consented to the risk of injuries involved in a particular act then
he can’t plead for damages under tort law if injury is caused. Therefore, ‘Volenti Non Fit Injuria’ is a
general defence to escape liability under tort law.
According to the present facts Rajan himself willingly (Voluntarily) consented to the risk of death
involved in the job that was explicitly communicated to him by the owner of the Circus. Hence, Rajan
Voluntarily gave an express consent to face all the injuries involved and hence no Injury caused to him
(for which he consented) can be an ‘Injuria’, i.e. ‘legal injury’, and no cause of action arises in tort law
unless there is a legal Injury (Injuria).
Thus, Mr. X will successfully claim the defence of ‘Volenti Non Fit Injuria’ and couldn’t be held liable to
pay damages under tort law.
Option (A) is an absurd statement, because vicarious liability is based on, Employer-Employee, Master-
Servant or Principal-Agent relationship and no such relationship is present between Mr. X and lion (an
animal can never be an employee or Servant or Agent).
Option (B) is a general statement which otherwise may be true but in context of the above Principle is
not a correct answer.
Option (C) gives a wrong reason for the right answer because the lion was an integral part of the Circus
owned by Mr. X and therefore, was not an independent agent.
Hence (D) is the correct answer.
135. Principle: Property can be transferred only by a living person to another living person.
Facts: A transfers a property of which he is the owner in favor of the unborn child of B.
136. Principle: A Contingent Contract is one which is enforceable only on happening of an uncertain
future event.
Facts: A entered into a Contract with B that if Salman’s upcoming movie ‘Bharat’ will earn more than
100 crore rupees than he will gift a brand new Car to B.
A. It will become enforceable only when Salman’s movie will earn 100 crore rupees
B. It is a valid contract and is enforceable from the moment it is entered in by the parties
C. It is an invalid Contract as it includes an element of chance or luck
D. A should be prosecuted for gambling
Correct answer: (A)
Explanation: The following agreement between A and B is a contingent contract because it can only be
enforced on happening of an uncertain future event (Salman’s Movie may or may not earn 100 crores).
The word ‘contingent’ literally means ‘on happening of an uncertain future event’.
A promised B that he will gift him the car only if the movie earned more than 100 crore rupees and it is
not a certain even, the contract can’t be enforced by B until and unless the movie earns 100 Crore
rupees (Uncertain Future event), and the agreement is thus a contingent Contract.
Option (B) is false because the nature of the Contract is Contingent.
Option (C) is irrelevant to the above Principle.
Option (D) is an independent statement, not having any sort of relation with the above Principle.
137. Principal: An agreement which is enforceable by law at the option of one or more of the parties
thereto, but not at the option of the other or others, is a voidable contract.
Facts: A and B were siblings and joint owner of their Family business. C who was their nearest
Competitor in the market with a view to establish his monopoly in the market made a contract with A in
writing to purchase their family business. However, Consent of A was obtained through fraud and B
never gave his consent.
Is it a valid contract?
A. It is a void contract.
B. It is a voidable Contract.
C. It is a valid Contract.
D. It is only an agreement and not a contract.
Correct Answer: (B)
Explanation: Section 19 of the Indian Contract act, 1872 states that a Contract in which Consent is
obtained through fraud is a voidable Contract at the behest of the Party whose consent is so caused. In
the present case it is expressly laid down that consent of A was obtained by C through fraud, making the
Contract Voidable.
Option (A) is wrong.
Option (C) is false because ‘Free consent’ which is one of the essential of a valid contract is missing.
Option (D) is not absolutely true as its enforcement depends upon the option of A and moreover the
assertion is an independent viewpoint not relatable to the principle.
Hence, (B) is the correct answer.
138. Principle: An agent having an authority to do an act has authority to do every lawful thing which is
necessary in order to do such act. An agent having an authority to carry on a business has authority to
do every lawful thing necessary for the purpose, or usually done in the course, of conducting such
business.
Facts: X is a money-lender residing in London. He employs Y to recover his debts due to Z living in Delhi.
Y in order to recover the debt on behalf of X contacts Z and demands the money back from him but was
refused on the ground that he will return the money only to X and not to any other person.
A. Y is an agent of X and can take any lawful step in order to recover the money from Z
B. Y has no authority to recover money from Z
C. Only X can recover money
D. Z can sue Y for Extortion
Correct Answer: (A)
Explanation: Y is a rightful agent of X in connection with the job of recovery of Debt from Z because X
has hired him for the above purpose and he has all the authority of an agent as stated in the principle
which is taken from section 188 of Indian Contract act 1872. Therefore, Y can take any legal step on
behalf of X in furtherance of recovering the debt from Z.
Option (B) is false because the Principle itself gives the Authority to Y (being an Agent) to do every lawful
thing which is necessary in order to Recover the debt.
Option (C) is absurd statement in reference to the above Principle.
Option (D) is related to Criminal law while the principle is from Contract law, thus eliminating it.
Hence, (A) is the correct answer.
139. Principle: In Indian Penal Code, Animals can commit no offence but offence can be committed
against them.
Facts: Salman is a well-known Superstar of the Bollywood industry. One day while he was shooting for
his upcoming movie, a Horse suddenly came there and caused eruption in the shooting of the Movie
scene. Salman got highly aggressed by it, and took out his shotgun from his car and killed the horse by
shooting him down. The Public got terrorized by his extreme act of violence, thus causing mischief.
A. Salman has committed no offence because his act of killing the horse was in his exercise of right
of Private defence.
B. Salman is a heinous man with no humanity and should be beaten by the Public
C. Salman has committed an offence under IPC, 1860
D. Salman has committed no offence because it was against an Animal and not against any human
Correct answer: (C)
Explanation: The above Principle states that Animals can never be accused in a Crime but they can be
victim of a Crime. In simple words, Human being is an essential ingredient to constitute a crime but
unlike the wrongdoer (who should always be a human) the victim of that Criminal act can also be an
animal.
In the current facts, Salman (Human) killed the horse (animal), which caused mischief among the public,
thus constituting an offence under Section 429 of IPC, 1860. All the conditions laid down in the Principle
are fulfilled and hence an offence under IPC is committed by Salman.
Option (A) is false because the facts clearly indicate that Salman killed the horse out of rage and anger
and not in order to protect him and others as the Horse didn’t possessed any threat to anyone’s life or
Property.
Option (B) is an absurd statement not backed by any rational legal viewpoint.
Option (D) is false in context of the Principle which clearly embarks that offence can be committed
against an animal.
Hence (C) is the correct answer.
Facts: A desires a Court to give judgment that B, C and D shall be punished for a crime which A says B, C
and D have committed.
(A) A must prove that B, C and D were present at the place of crime
(B) A must prove that B, C and D have committed the crime
(C) B, C and D must prove that they have not committed the crime
(D) Police must prove that B, C and D have committed the crime
141. Principle: Par in parem non habit Imperium, meaning one state can’t exercise jurisdiction over
another state.
Facts: King of Bhutan visited India and stayed there for one Month. During his stay, he fell in love with
an Indian girl named Chutki. He promised Chutki that he will marry her, but later on breached that
promise and went back to his Country.
Consequently, Chutki sued him for Breach of trust.
Decide:
A. Chutki will succeed, because everyone is equal before law, whether a king or a beggar.
B. Chutki will not succeed because he is a Sovereign of another nation
C. Chutki will succeed because the king breached her trust
D. Chutki, can’t succeed because he was not an Indian Citizen.
Correct answer: (B)
Explanation: The Principles of International law states that a state/nation can’t exercise any jurisdiction
over the sovereign of another State/Nation. The above Principle also reflects the same thing and means
that one Sovereign can’t exercise any jurisdiction of its laws over a Sovereign of another nation (neither
civil nor Criminal). The idea behind this Principle is that a Person who is a Sovereign, i.e. Source of law in
his nation can’t be adjudicated by any laws of other country.
The king of Bhutan is a Sovereign and hence, Indian Judiciary can’t exercise any Jurisdiction over him,
therefore, Chutki can’t succeed.
It is to be noted that in a democracy like India no person is a sovereign and the Sovereignty lies in the
Written Constitution.
Option (A) is a principle of natural justice, but incorrect in accordance to the Principle stated above.
Option (C) is incorrect due to the immunity given to the king under the above Principle.
Option (D) is absurd, because even a Foreigner can be prosecuted for committing a crime in India or
against an Indian citizen.
Hence, (B) is the correct answer.
142. Principle: No person shall be prosecuted and Punished for the same offence, more than once.
Fact: Nandu was charged for the offence of Rape, under Section 376 of IPC, 1860, but he was finally
acquitted by the Apex court due to lack of evidence against him. The judgement led to a huge Public
outrage all over the nation, and resultantly the case was reopened again. In turn, Nandu Bought a Writ
petition in the Supreme Court of India under Article 32 of the Constitution of India, challenging the move
to be in violation of his Fundamental Right enshrined under Article 20(2).
A. Nandu will succeed, because reopening the case is violative of doctrine of Double Jeopardy
which is a Fundamental right, given in our Constitution.
B. Nandu will succeed, because he was already acquitted of all the charges and reopening the case
is abusive to the Criminal Justice system of India.
C. Nandu will succeed because Indian Judiciary already has a lot of burden of pending cases over it,
and reopening an already decided case is not a good idea.
D. Nandu will not succeed because none of his Fundamental Right is violated.
Explanation: A writ Petition in Supreme Court under Article 32 of the Constitution is maintainable only if
any of the Right enshrined in Part III (Fundamental right) of the constitution is violated of any person,
entitled to that right by the State and its Machineries. Article 20(2) is a Fundamental right for the
citizens of India in respect of Protection from ‘conviction for offences’. It lays down that ‘No person shall
be prosecuted and Punished for the same offence, more than once’. So, the necessary element for
successfully claiming protection under this Article is that, a person should be Prosecuted as well as
punished (Convicted) and, it is not available to a person who is Prosecuted and Acquitted. The ‘Doctrine
of Double Jeopardy’ is narrower in Indian Constitution as compared to the American constitution. Only
the Principle of ‘Autrefois Convict’ is applicable to Article 20(2) of the Constitution of India, while in
American Constitution both ‘Autrefois Convict’ as well as ‘Autrefois acquit’ are applicable.
Since, Nandu was only prosecuted but not convicted for the offence, that’s why reopening the case is
not violating his Fundamental right under Article 20(2).
Option (A) is false; because the Doctrine of Double Jeopardy in Indian Constitution has a narrower scope
and is only limited to ‘Autrefois Convict’ and Nandu’s case comes under ‘Autrefois Acquit’.
Option (C) is just a personal opinion and is not backed by any legal provision; moreover it is not related
in any manner with the above principle.
[Definition:
Autrefois Acquit: This Principle lays that, once a person is prosecuted and Acquitted, he can’t be
prosecuted again.
Autrefois Convict: This Principle states that once a person is Prosecuted and Punished (Convicted) he
can’t be prosecuted again.]
143. Principle: Civil Suit can be filed where defendant resides or carries on business or where cause of
action arises.
Facts: X is a trader of Cricket Goods based in Meerut. ABC cricket Academy, New Delhi had placed a
large order of 50 Cricket kits to him and also paid him an advance of 10,000 Rupees in a meeting at
Ghaziabad. In return X had promised in writing to deliver the goods by 15 th March, 2019 but didn’t
deliver the goods till 30th March, 2019, due to which the Academy incurred huge losses.
In order to recover the amount from X, ABC Cricket academy decided to sue him for breach of contract.
Where the Academy can file the civil suit?
A. Anywhere in India.
B. Only in Meerut.
C. Only in Ghaziabad or New Delhi.
D. Either in Meerut or Ghaziabad.
Explanation: According to the Principle a civil suit can be filed at any of the three places, namely:
(i) Where Defendant Resides;
(ii) Where the defendant carries on his business; or
(iii) Where cause of action arises.
In the above facts X is the defendant so ABC ltd. can file civil suit against him either in Meerut (where
the defendant resides and carries on Business) or in Ghaziabad (where cause of action, i.e. the Contract
between the two arose).
Option (B) is not completely right as Meerut is not the only option.
Option (C) is false because at New Delhi, neither cause of action arose, nor the defendant resides or
carries on business.
144. Principle: An Employer is liable for the wrongful act of his employee acting within the course of
employment.
Facts: Ramesh was an employee in the Noida branch of IDBI bank, his neighbour Seema had a bank
account in that bank. One day Seema wanted to deposit 50,000 Rupees cash in her bank account, as she
was planning to leave for the bank she saw Ramesh and gave him the money to get it deposited in her
account. Instead of depositing the money in her account, Ramesh took the money and Ran away. When
Seema got to know about it, she sued the Bank for recovery of damages under the principle of Vicarious
Liability.
A. The Bank should be held liable for the wrongful act of Ramesh, who was its Employee.
B. The Bank can’t be held liable for the Wrongful act of Ramesh.
D. Seema was herself at fault and should not expect any legal remedy.
Explanation: The principle of vicarious liability makes the Employer liable only for those wrongful acts of
its employee done within the course of Employment and within the capacity of being an Employee. In
the present case Ramesh received money from Seema not in the capacity of an Employee of the bank
but in his personal capacity of being Seema’s Neighbour. Hence, Ramesh was not acting in the course of
Employment of IDBI bank, thus eliminating the liability of the bank.
Option (A) is wrong in the factual situation of the present case because Ramesh was not in capacity of
the bank’s employee when he committed the wrong.
Option (C) is true and also correct but it is not relatable to the Principle stated.
145. Principle: The Constitution provides for a High court for each state.
Fact: The State of Punjab and Haryana have a joint High Court at Chandigarh and not their own
independent high courts within their territory.
Explanation: Article 214 of the Constitution of India lays down that ‘there should be a high court for
each state’. It doesn’t necessarily mean that there should be a high court in each state. For instance,
states of Assam, Nagaland, Arunachal Pradesh and Mizoram shares a Common high Court (Guwahati
High Court). So, it means that a state should come under the Jurisdiction of a high court but it is not
compulsory that it (high court) should be situated within the geographical limits of that State.
Facts: Rohit got sticked to a live electricity wire and was about to die due to electricity shock, when
Manoj saw him and came to his rescue. In order to save him, Manoj hit him hardly by a wooden Rod due
to which Rohit’s Ribs got fractured, but eventually his life was saved. Instead of thanking him, Rohit
bought a suit against him for recovery of damages for grievously hurting him.
A. Manoj is liable because his act was unlawful and deserves to be punished.
B. Manoj is not liable because the act was done in good faith and in order to save Rohit’s life.
C. Rohit should be prosecuted for attempting to commit Suicide.
D. Manoj is liable as in no circumstance he can have a right to injure someone.
Explanation: Manoj’s act of hitting Rohit was a ‘Bonafide act’ and it was done with the intention of
saving Rohit’s Life, i.e. in order to prevent a bigger harm. In tort law, any harm which is caused with
bonafide intention to prevent a bigger harm is a complete defence to escape tortious liability. Therefore,
Manoj can’t be sued by Rohit under tort law.
Option (A) is wrong because the act conforms to the exercise of Manoj’s Lawful duty.
Option (C), might be correct but it is related to Criminal law, while the principle is from tort law, hence it
is irrelevant in the context of the question.
Facts: Hitesh, who is a 6 years and 11 months old child and son of a police officer, entered into a quarrel
with one of his classmate at school and threatened him of dire consequences. Next day he took the
service Revolver of his father to school and brutally shot down his classmate in his head and heart like a
dreaded criminal without showing any mercy.
Explanation: Section 82 of the IPC, 1860 lays down that ‘nothing is an offence which is done by a child
under seven years of age’. Therefore, Hitesh, who is below 7 years of age can’t be held culpable for any
offence, irrespective of the fact that, whatever is the manner of commission of crime or whether he
understood the consequences of his act or not. In case of a Child below 7 years of age such measures
are irrelevant.
Option (B) is true but can’t be connected with the Principle stated above.
Option (C) may also be true but not correct in context of the Principle.
148. Principle: Whoever dishonestly takes away any movable property out of the possession of any
other person without that person’s consent, moves that property in order to such taking, is said to
commit theft.
Facts: X was a renowned thief of his area and he had many stolen items at his home. One day, Y entered
X’s house during afternoon hours when X was sleeping and silently took out a Mobile Phone from X’s
Pocket that X had previously stolen from somewhere else.
Decide:
A. Y has committed theft against X, the moment he took out the mobile phone from his pocket.
B. Y has not committed theft because X was not the legal owner of that mobile Phone.
C. A thief has no right to prosecute others for theft.
D. X should be punished for Theft and not Y.
Explanation: The definition of theft given in the Principle is taken from Section 378 of IPC, 1860. On a
close look at the language of this definition it can be inferred that the offence of theft is against
possession and not against ownership. ‘Possession’ means physical presence of a thing into a person’s
custody, it is immaterial whether it is legal or illegal. On the other hand ‘Ownership’ is a person’s right
over a thing at the exclusion of all the other members of the society, and it is always legal or lawful.
Since, theft is an offence against possession which is not necessarily required to be lawful, that’s why a
theft can be committed even against a thief for theft of a stolen item. All the necessary elements of
theft are fulfilled by the act of Y.
Option (B) is not correct because ownership is immaterial in the offence of theft.
Option (C) is false. A thief or any other criminal can also sought legal action if any crime (including theft)
is committed against him.
Option (D) is not completely true, because Y has also committed the same offence as X.
149. Principle: Where the death of a woman is caused by any burns or bodily injury or occurs otherwise
than under normal circumstances within seven years of her marriage and it is shown that soon before
her death she was subjected to cruelty or harassment by her husband or any relative of her husband for,
or in connection with, any demand for dowry, such death shall be called “dowry death”, and such
husband or relative shall be deemed to have caused her death.
Facts: Ragini was a poor girl who hailed from an extremely backward family. During College life she fell
in love with a boy named Vishal who belonged to a Rich family of Bureaucrats, and later, on 22.01.2010
both married each other against the wishes of their Families. After 2 years of their Marriage, the couple
had a son. After Hearing this news that they have become Grandparents, the Parents of Vishal accepted
the couple and took them to their house. For some time they all lived joyfully as a happy family but later
on her in laws started harassing her for dowry and for being a girl from a lower caste. Finally on
13.01.2017 she was found dead in her house under suspicious circumstances.
Explanation: All the ingredients that constitute the offence of dowry death under Section 304B of IPC,
1860 are fulfilled in the facts given above, hence her husband and in laws could be booked for the
offence of dowry death.
Option (B) might be true but not definitely correct as it depends upon the circumstantial evidences.
Option (C) is incorrect because nowhere in the facts it is mentioned that she committed suicide.
Option (D) is irrelevant because Dowry death and Murder looks similar but are different offences.
150. Principle: Human being is one of the most essential ingredients to constitute a Crime.
Facts: Raju had a dog named Tommy. Tommy was exceptionally intelligent and he understood almost all
Human Communications and feelings. One day when Raju was playing with tommy in a Park, Ratan, who
was one of the Creditor of Raju, came there and started abusing him in front of everyone for not
returning his money on time. Aggrieved by all this, Tommy started barking loudly and attacked Ratan,
biting him on his Stomach and Legs. Consequently, Ratan was grievously injured and was admitted to a
Hospital.
Ratan decided to lodge an FIR against Tommy for grievously hurting him.
A. Ratan will succeed as tommy had committed a crime under Section 320 of IPC.
B. Ratan will succeed because Tommy had the necessary Mens Rea as he had enough intelligence
to understand Human Communications and feelings.
C. Tommy can’t be prosecuted because he is not a Human.
D. Tommy can’t be prosecuted because he was justified in his act.
(i) Human Being; (ii) Mens Rea, i.e. Guilty Mind; (iii) Actus Reus, i.e. Criminal Act; (iv) Injury caused to
another person.
Since, Tommy is a dog and not a Human being that’s why any act committed by him can’t be a Crime and
thus, he can’t be prosecuted in a law court.
Option (A) is incorrect as the offence of Grievous hurt under Section 320 of IPC can be committed only a
Human and not an Animal.
Principle 2: Right to Freedom of Speech and expression is a Fundamental Right of every citizen, subject
to reasonable restrictions.
Facts: Manto is a well-known writer of his country. He is recognized all over the world for his literature
related to women. One of his short stories named ‘Thanda-Ghosht’ portrayed a sub-urban place set up
in the background of the Indo-Pak Partition days. The Central theme of the whole story was particularly
Human lust. It revolved around the life of a Hindu man who satisfied his demonic lust by having a carnal
intercourse with a Corpse of a Muslim girl. The words used in the whole story were highly adulterous.
A. Manto should be punished for the offence of Obscenity under section 292 of IPC.
B. Manto has committed no offence because he has just exercised his Fundamental right given
under Article 19(1) (a) of the Constitution of India.
C. Manto has committed the offence of obscenity because he has no freedom of speech and
expression as he migrated to Pakistan and was no more an Indian Citizen.
D. Manto should not be punished because he never published his writing.
Correct Answer: (A)
Explanation: Section 292 of IPC, 1860 defines and lays down punishment for the offence of Obscenity.
As per the facts of this question all the requirements in order to constitute the offence of Obscenity are
fulfilled. ‘Obscenity’ refers to sexual words, either spoken or written and acts which shock People and
society at large. Therefore, the basic requirement for this offence is that the literature should be
published, i.e. should come in the knowledge of any third person from the public.
The Second Principle lays down the fundamental right given to the Indian Citizens by our Constitution
under Article 19(1) (a), but this fundamental right can’t safeguard Manto from being Prosecuted and
Punished under Section 292 of IPC because this Right is subject to reasonable restrictions (including
Public Order and Morality). Since, ‘Thanda-Ghosht’ was against morality it came within the purview of
reasonable restrictions to Article 19(1) (a). Therefore, Manto should be punished for the offence of
Obscenity.
Option (B) is false because the right is not absolute and had to be exercised subject to reasonable
restrictions.
Option (C) can’t be said to be absolutely true because no further information is provided regarding his
citizenship.
Option (D) is wrong because the facts stated above are inadequate to prove it absolutely right that he
never published his writing. Moreover, from the facts it can be inferred impliedly that he used to publish
his writings (that’s why he was a well-known writer).
152. Principle 1: Culpable homicide is genus and Murder is one of its species.
Principle 2: Whoever Commits Murder should be punished with death or Imprisonment for life.
Facts: Raj was travelling to his hometown by his car. The Highway was clear and in order to reach home
soon he speeded up his car to 150kmph, while the prescribed speed limit on that highway was 120kmph
as indicated at various boards and hoarding on that highway. Suddenly, a Man named Shyam who was
heavily drunk came in front of Raj’s car and the car due to such a high speed got out of control and hit
Shyam, who died on the spot.
Explanation: Raj was driving negligently as he didn’t obey the prescribed speed limit that was indicated
on various boards at the Highway itself. ‘Negligence’ means lack of proper care, but it is to be noted that
Mens Rea (Guilty Intention) is absent in cases of Negligence, that’s the reason why crimes related to
negligence have lower degree punishment. In the present case Raj didn’t had any intention to kill Shyam
and everything that happened was due to his ‘lack of proper care’ (Negligence). Hence, he should be
prosecuted under section 304A of IPC which is related to causing death due to rash and negligent act.
The Actus Reus (Criminal act) is same in both, murder (Section 300) and Death caused by Negligence
(Section 304A) as both are culpable homicide but the former is punished with death or imprisonment for
life while the punishment prescribed for the latter is just a maximum imprisonment of 2 years. The
difference between the two is due to Mens Rea, which is present in former but not in the latter.
Option (A) is wrong because Murder is an intentional act according to its definition under Section 300 of
IPC but Raj’s act was unintentional.
Option (D) is not correct because an accused can’t take the defence of Victim’s fault to escape from
Criminal liability if he was also at fault.
Principle 2: The offence punishable under section 354D of IPC, 1860 is gender specific.
Facts: Ritesh and Raju are 3rd year graduation students and classmates as well. They both had a good
tuning with each other. Raju started getting attracted towards Ritesh and loved to be in his company,
hence, wherever Ritesh went, Raju followed. Fed-up by all this, Ritesh started ignoring Raju but all his
attempts went in vain and Raju never left him alone. Frustrated by all this, Ritesh finally decided to lodge
an FIR against Raju for stalking him.
Decide:
Explanation: Section 354 D of IPC, 1860 defines stalking and it expressly mentions that Stalking is a
Gender Specific offence and can be committed only by a male against a female and not against a male.
Since, Ritesh is a male, that’s why Stalking can’t be committed against him.
Option (B) is wrong because despite the fact that the actions of Raju fulfill all the requirement of
Stalking, he can’t be prosecuted as his actions were against a male and not a female.
Option (C) might be true but not related to the Principles stated above in any manner.
154. Principle: When a criminal act is done by several persons in furtherance of the common intention
of all, each of such persons is liable for that act in the same manner as if it were done by him alone.
Facts: A, B, C and D decided to commit Robbery at Mr. Z’s house, who was a renowned businessman of
the village. After planning and preparing for the crime, they decided to execute their plan and went to
Mr. Z’s home. According to their Plan, A decided to stop outside the premises of Mr. Z and awaited for
rest of his gang members to arrive back and also kept a watch outside the house in order to ensure that
everything was fine, while B, C and D went inside and attacked Mr. Z and his wife and tied them with a
Rope and started Robbing the Valuables present in the house. Suddenly, Ramu, who was servant of Mr.
Z came from behind and started shouting. Panicked by it, B attacked Ramu with a Knife leading to his
death.
When Prosecuted in the Court A pleaded that he was involved neither in Murder nor in robbery as he
was not physically involved in neither of the two acts. C and D pleaded before the court to be guilty for
Robbery but not for Murder of Ramu because it was committed by B and they didn’t have any intention
to kill him.
Decide:
A. A should be acquitted of all charges, while B, C and D should be convicted for Murder and
Robbery.
B. All of them should be prosecuted for Robbery, but for Murder of Ramu only B should be
prosecuted.
C. A should be acquitted of all charges, while B, C and D should be convicted for Robbery jointly
and for Murder B should be separately prosecuted.
D. All four of them should be prosecuted for Robbery as well as Murder jointly and severely.
Explanation: The above Principle is taken from Section 34 of IPC which states about ‘Common
Intention’. The definition simply indicates that if a group of persons (more than one person) jointly
commits any crime with a common intention or even if any one of them commits it in furtherance of
common intention of all then each person of the group will be liable for the whole crime and all offences
done in pursuance of that crime. So, the above principle imposes joint and severe liability upon each
member of a group. Therefore, all of the four should be prosecuted for Committing robbery under
section 392 of IPC and Murdering Ramu under section 302 of IPC because the Murder of Ramu was
committed in furtherance of the crime (Robbery) for which they all had a common intention.
Option (A) is false because according to the Principle stated above physical involvement is not a
necessary requirement and mere Common intention is enough to held guilt of the accused.
Option (B) is incorrect because the Murder of Ramu was in furtherance of the crime for which they all
had intended.
Option (C) is wrong as there is no joint and severe liability as provided in the Principle.
155. Principle 1: Sedition is an offence, which involves inciting hatred feelings against the State.
Facts: Kanhaiya was a bright student and a student leader of his university. He used to be actively
involved in all the political matters of his reach. One day while addressing a large public gathering at the
University Campus he started praising some Terrorists who were accused of committing and planning
genocides in India, but were not yet convicted. Furthermore he shouted slogans like ‘Long live Pakistan’.
Decide:
A. Kanhaiya has not committed Sedition because he has not said anything against the Government.
B. Kanhaiya has committed sedition because he has praised ‘Pakistan’ which is an enemy Country
of India.
C. Kanhaiya had just expressed his personal beliefs and is in no way offensive to the State.
D. Kanhaiya should be punished for preparation to commit sedition.
Facts: Harpreet was a girl of 15 years of age when she was attacked by Acid by some unknown
assailants. As a consequence of the attack she got 95% burns and before dying in the hospital she named
Dinesh, a senior student in her school to be the person who attacked her. In police investigation no
concrete evidence was found against Dinesh and he was acquitted by the district court of all charges. On
an appeal to the Hon’ble high Court it was argued by the Public Prosecutor that Dinesh was Guilty for
the offence beyond any reasonable doubt because of the dying declaration made by the victim herself
and no other corroboration of evidence was required.
A. The High court should convict Dinesh for murdering Harpreet and reverse the order of District
Court.
B. The high Court should not uphold the order of district court.
C. Benefit of doubt should be given to the accused
D. The Police might be bribed by the accused.
Explanation: According to section 32(1) of the Indian Evidence Act, a victim’s statement on death bed
should be deemed to be completely true and no corroboration or support of any other evidence is
required in order to hold guilt of the accused. Since, Harpreet explicitly named and identified Dinesh in
her dying declaration under section 32(1) of Indian Evidence act, that’s why Dinesh’s guilt is proved
beyond any reasonable doubt. Therefore, the high court should reverse the lower court order and
convict Dinesh for Murdering Harpreet.
Option (B) might be true but not correct in reference to the above Principle.
Option (D) is a mere Opinion, which is irrelevant in the context of the Principle.
Explanation: According to the above facts the statement made by Kanhaiya in the Public gathering was
in no way seditious. Section 124A of IPC defines Sedition as: “Whoever, by words, either spoken or
written, or by signs, or by visible representation, or otherwise, brings or attempts to bring into hatred or
contempt, or excites or attempts to excite disaffection towards, the Government established by law in
India commits the offence of Sedition”.
The facts nowhere fulfill the requirement for constituting the offence of sedition as given in the above
principle. The Terrorists, whom Kanhaiya had praised, were not yet convicted and were just accused,
and due to the principle of ‘Presumption of innocence until proven Guilty’ they can’t be adjudged as
enemies of the Nation (State). Moreover, India and Pakistan are not in a state of war, neither they both
are proclaimed enemy nations, that’s why Chanting slogans in honor of Pakistan isn’t seditious against
the Government of India. So, such statement can neither incite nor it is made with intention to incite
feeling of hatred against the state, i.e. Government of India.
Option (B) is a wrong and gives a false reason as there had been no official announcement of enmity
between the two nations.
157. Principle: Oral Evidence must always be direct and not hearsay.
Facts: Suresh was a Prosecution witness in a Kidnapping case against Mahesh, who was his good friend.
Suresh told the court that 6 months ago Mahesh told him, his intention to kidnap the Victim.
Explanation: The Principle stated in the question says that an Oral (Spoken) evidence must be direct
and not hearsay. ‘Direct’ means that the person making a statement had seen it by his own eyes. On the
other hand ‘hearsay’ means what a person had heard and is not witnessed by his own eyes.
Whatever, Suresh told the court was not seen or witnessed by him and he just narrated what he heard
from another person (Mahesh). Therefore, it can be clearly concluded that the oral statement made by
Suresh was ‘Hearsay’ and not ‘Direct’, and hence not admissible in a court of law according to the
Principle.
Option (A) seems to be true but can’t be the correct answer when the Principle is applied to the fact.
Fact: A’s five year old son was lost in a fair. In addition to lodging a Missing report in the Police Station,
he also posted an advertisement in a newspaper, declaring that whoever will find his son will be
rewarded with 50,000 rupees. X, who was an unemployed youth, read the advertisement and identified
it as a great opportunity to earn money as he was facing a great economic crisis. Therefore, he started
finding the boy and luckily found him and took him to his father, but unfortunately X was denied by A to
be paid as per the terms of the advertisement.
A. There is no contract between the two because there is no communication of acceptance and
proposal.
B. There is no Contract because proposal was not made by A to X
C. There is a void Contract between the two because there is no consideration.
D. A has breached the contract and it was a valid Contract, because in such offers performance
amounts to acceptance.
Explanation: The Proposal/offer made by A in the newspaper was made to the public at large, i.e. to all
the members of the society who read it. Hence, it was a general offer/proposal. According to the
Principle a general offer can be accepted by Performance, therefore in case of general offer
communication of acceptance to the promisor is not required and mere performance as per the terms
and conditions of the offer is enough.
A proposal when accepted becomes a Promise and a Promise when comes with a consideration
becomes an agreement and an agreement with a lawful object is a Contract. In the above case Proposal
is made by A (in the newspaper), and it is accepted by X (by performance) and consideration is also
present (50,000 Rupees), as well as the object is lawful. Thus, it is a valid Contract and X can successfully
sue A for Breach of Contract.
Option (A) is wrong as Communication of Proposal (in newspaper) is present and communication of
acceptance is not required in General offers.
Option (B) is incorrect because proposal was made by A to the General Public (including X).
159. Principle: Husband and wife are deemed in law to be a single economic unit.
Fact: A is the spouse of B, and both are married for more than a decade. A’s wife B borrowed some
money from one of A’s friend without his prior knowledge and gave that money to her brother. Later on
his friend demanded that money back from A but he refused to pay citing the reason that he was not
the one to whom he had granted the credit and should demand it back from B only.
Explanation: On a simple application of the above principle to the facts it can be concluded that
A(Husband) is liable to pay back the amount borrowed by B (Wife) because in law both are deemed to
be single economic unit. It means that the liabilities of A are liability of B too and vice-versa is also true.
The fact that the borrowed funds were used by a third person (B’s brother) and not by the couple
doesn’t make any difference to the answer.
Option (B) is part true and part false as it is undoubtedly true that they are separate legal entity but
economically, i.e. in matters of money they are one entity, making both of them jointly liable for each
other’s borrowings.
Facts: XYZ ltd. is the most successful company in the flourishing Chemical Industry in India. It had always
fulfilled all its corporate responsibilities and had organized various Environmental protection Programs,
like tree plantation week, Save Water campaign etc. On the fateful night of 2 nd December, 1984 a slight
Earthquake came due to which a vast container present in the factory of XYZ ltd. that contained toxic
chemical substances broke down and those chemicals submerged into the water of nearby River.
Resultantly, the River Water and all the underground water became poisonous and hundreds of people
lost their life. The Supreme Court of India took suo Motu Cognizance and sued the directors of XYZ ltd.
for the payment of exemplary damages to the victim’s and their families.
The lawyer of the Company contended that the accident was beyond Human Control and was a result of
the Earthquake which was an act of God, and no negligence on part of the company was present.
Decide:
A. XYZ ltd. can’t be held liable as the death were direct result of the Earthquake.
B. XYZ ltd. should be held absolutely liable and therefore no defence should be available to it.
C. XYZ ltd. should be held liable because the death were a direct result of the mixing of their
Chemicals into the river water and not due to Earthquake, which was an act of God.
D. XYZ ltd. had not been negligent on its part, therefore it can’t be held liable.
Explanation: The Principle of Absolute liability states that ‘if an industry or enterprise is engaged in
some inherently dangerous activity from which it is deriving commercial gain and that activity is capable
of causing catastrophic damage then the industry officials are absolutely liable to pay compensation to
the aggrieved parties. The Corporate can’t plead that there was no negligence on their part neither they
can successfully claim any of the defences available in tort law.’ According to the facts of the above case
XYZ ltd. should be held liable under the principle of absolute liability, therefore, the general defence of
Act of God is also unavailable to it in order to escape tortious liability. All the essentials required to be
held liable under the principle of Absolute liability are fulfilled by XYZ ltd.
Option (A) is false because the deaths were direct result of the toxic chemicals that got mixed up in the
River water and not were directly caused by Earthquake.
Option (C) is true but the reason is not correct because even an ‘Act of god’ is not an available defence
in Absolute liability.